Sunteți pe pagina 1din 82

THE FIBONACCI QUARTERLY

THE OFFICIAL JOURNAL OF

THE FIBONACCI ASSOCIATION

VOLUME 3

IffiST

NUMBER 1

PART 1 A D V A N C E D

A NEW CHARACTERIZATION OF THE FIBONACCI NUMBERS . . . . .


SOME RESULTS CONCERNING POLYOMINOES

J.L. Brown, Jr.

. . . . . David A. Klamer

ON THE REPRESENTATION OF INTEGERS AS SUMS OF DISTINCT


FIBONACCI NUMBERS
ON A GENERAL FIBONACCI IDENTITY

ADVANCED PROBLEMS AND SOLUTIONS

H.H. Ferns

21

John H. Halton

31

Edited by Vemer E. Hoggatt, Jr.

PART

44

IIELEMENTARY

PRODUCTS OF ODDS

Sheryl B. Tadlock

54

EXPLORING SCALENE FIBONACCI POLYGONS

. .

Peck

57

Brother U. Alfred

59

NOTE ON THIRD ORDER DETERMINANTS


MORE ON FIBONACCI NIM

. . . .

. . . . . . . .

Jeremy

C. Pond and Donald

ON SUMS F ^ F 2
x
y
PHYLLOTAXIS

.-.

LETTER TO THE EDITOR

FEBRUARY

. . . . . . . .

F. Howells

. Benjamin

61

Sharpe

63

Karchmar

64

Marjorie Bicknell

67

E.J.

FIBONACCI FANTASY: THE SQUARE ROOT OF THE Q MATRIX . . . .

ELEMENTARY PROBLEMS AND SOLUTIONS

C.B.A.

P. Naor

71

Edited by A.P. Hillman

74

1965

THE FIBONACCI QUARTERLY


OFFICIAL ORGAN OF

THE FIBONACCI ASSOCIATION


A JOURNAL DEVOTED TO THE STUDY OE INTEGERS

WITH SPECIAL

PROPERTIES

EDITORIAL BOARD

B r o t h e r U. Alfred
H. L. Alder
S, L. Basin
John L. Brown, J r .
L. C a r l i t z
H. W. Gould
A. P . Hillman

V. E. Hoggatt, J r .
Donald E. Knuth
C. T. Long
Leo M o s e r
I. D. Ruggles
D. E. Thoro

WITH THE COOPERATION OF

P . M. Anselone
C h a r l e s H. King
T e r r y Brennan
L. H. Lange
Maxey Brooke
Douglas Lind
Paul F . Byrd
Calvin T. Long
Leonard C a r l i t z
J a m e s Maxwell
Calvin D. C r a b i l l
S i s t e r M. de Sales McNabb
H. W. Eves
C. D. Olds
John H. Halton
D. W. Robinson
R i c h a r d A. Hayes
A z r i e l Rosenfeld
A. F . H o r a d a m
John E 0 Vinson
Dov J a r d e n
Lloyd Walker
Stephen J e r b i c
C h a r l e s R. Wall
R0 P . Kelisky
The California M a t h e m a t i c s Council
All s u b s c r i p t i o n c o r r e s p o n d e n c e should be a d d r e s s e d to B r o t h e r U.
Alfred, St. M a r y ' s College, Calif. All checks ($4.00 p e r y e a r ) should
be m a d e out to the Fibonacci A s s o c i a t i o n or the Fibonacci Q u a r t e r l y .
M a n u s c r i p t s intended for publication in the Q u a r t e r l y should be sent
to V e r n e r E . Hoggatt, J r . , M a t h e m a t i c s D e p a r t m e n t , San J o s e . S t a t e
College, San J o s e , Calif. All m a n u s c r i p t s should be typed, doublespaced. Drawings should be m a d e the s a m e size a s they will a p p e a r
in the Q u a r t e r l y , and should be done in India ink on e i t h e r vellum or
bond p a p e r . Authors should keep a copy of the m a n u s c r i p t sent to the
editors.
The Q u a r t e r l y is e n t e r e d a s t h i r d c l a s s m a i l at the St. M a r y ' s College
P o s t Office, California^ a s an official publication of the Fibonacci
Association.

A NEW CHARACTERIZATION OF THE FIBONACCI NUMBERS


J.L. BROWN, JR.
The Pennsylvania State University

1.

INTRODUCTION

A t h e o r e m due t o E . Zeckendorf (see [1] for proof and discussion)


a s s e r t s that e v e r y positive i n t e g e r n can be r e p r e s e n t e d uniquely a s
a sum of distinct Fibonacci n u m b e r s such that no two consecutive F i b onacci n u m b e r s

a p p e a r in the r e p r e s e n t a t i o n .

With the definition

u, = 1, u~ = 2, and u . 0 = u ., + u for n 2 1, Zeckendorf' s t h e o r e m


1
2
n+2
n+1
n
yields an expansion for each positive i n t e g e r n in the form

n = 2
where

a.

a.u.

1
is e i t h e r 0 or 1 for each i >. 1 and

a.a.t,

Thus each positive i n t e g e r


a,9a?9a~,, . ., a . , . . . , w h e r e
a p p e a r s in the Zeckendorf
c o n s t r a i n t a.a.,,
- 0 for
l i+l
l ' s can n e v e r a p p e a r in the

= 0 for

i >L 1

i+l

n can be a s s o c i a t e d with a binary sequence


for given n, we see that a.(n) = 1 if u.
expansion of n; o t h e r w i s e a.(n) = 0. The
i _> 1 effectivelyJ s t a t e s that two consecutive
b i n a r y sequence c o r r e s p o n d i n g to n.

For

e x a m p l e , if n = 17, then 17 = 1 + 3 + 1 3 - u, + u~ + U/, so that 17 is


a s s o c i a t e d with the b i n a r y sequence 101001. (It is c l e a r that each such
expansion m u s t have all z e r o s to the right of some i = i

depending on

n and t h e s e noncontributing z e r o s a r e s u p p r e s s e d . )
The question a r i s e s a s to what o c c u r s if, i n s t e a d of disallowing
two consecutive n o n - z e r o coefficients in a Fibonacci expansion, we
disallow two consecutive z e r o coefficients,,

In other w o r d s , we wish

to c o n s i d e r r e p r e s e n t i n g an a r b i t r a r y positive i n t e g e r

n a s a sum of

d i s t i n c t Fibonacci n u m b e r s ,
N

n = 2 /3.u.
1
with b i n a r y coefficients
i = 1, 2, . . . , N - 2 .

satisfying

(3^ = 1 and

}, +

ft.

>. 1

for

In the following, T h e o r e m 1 affords a r e s u l t dual

A NEW CHARACTERIZATION OF

February

to the Zeckendorf t h e o r e m by showing that such an expansion always


e x i s t s for e v e r y positive i n t e g e r and m o r e o v e r the expansion is unique
under the i m p o s e d coefficient c o n s t r a i n t s .

T h e o r e m 2, which is our

m a i n r e s u l t , shows that the expansion p r o p e r t y of T h e o r e m 1 t o g e t h e r


with the uniqueness r e q u i r e m e n t is sufficient to c h a r a c t e r i z e the F i b onacci n u m b e r s .

This c o n v e r s e t h e o r e m for the dual r e p r e s e n t a t i o n

c o r r e s p o n d s to Daykin's r e s u l t [2] on the c o n v e r s e to

Zeckendorf's

theorem.

2.
Definition 1:

A DUAL-ZECKENDORF THEOREM

The F i b o n a c c i sequence

u-> = 2, and u . n = u ., + u for


2
n-2
n+1
n

/ u. \. is defined by

u, = 1,

n > 1.

L e m m a 1:
u

k+l

_ 1

+ U

k-2

+ u

k-4

+ u

+ U

dd-

+ U

k-2

+ u

k-4

' *

+ u

+ U

for

even

+ U

k-2

+ u

k-4

(a)
U

(b)

k+1 "

k+l

'

+ u

k-2p+2

+ u

k-2p+l

'

where
k
p = 1, 2, 0 . o , =- for
(c)

ic+l " ^

k-1
k even and p = 1 , 2 , . . . , -y for
k-1
"n"

is defined to be z e r o for
Proof.

2
1

- fr

k > 1,

where

k odd.

X
m

n < m.

The s t r a i g h t f o r w a r d inductive proof is left to the r e a d e r .


Our f i r s t t h e o r e m , as noted in the introduction, is e s s e n t i a l l y a

dual of the Zeckendorf t h e o r e m [l] :


T h e o r e m 1:
E v e r y positive i n t e g e r n h a s one and only one r e p r e s e n tation in the form
k
(1)
n = X jS.u. ,
1
w h e r e the /3. a r e b i n a r y digits satisfying

1965

THE FIBONACCI NUMBERS

(2)

^ i ^ i + 1 ^ 1 *or i = 1,2, . . . ,

k-2

and
(3)

/3k

= 1

F o r a given positive i n t e g e r

n, the value of k is d e t e r m i n e d a s

the unique i n t e g e r for which the inequality,


(4)

uk+1 - 2

<n l u

k + 2

- 2

is satisfied.
Convention:

It will be a s s u m e d without explicit mention in the r e -

m a i n d e r of the p a p e r that all expansion coefficients ( s u b s c r i p t e d v a r iables

a and j3 ) a r e b i n a r y digits, that i s , digits having e i t h e r the

value z e r o or the value unity.


Proof of T h e o r e m :
(4).

Let n be a positive i n t e g e r satisfying inequality

F r o m (c) of L e m m a 1, we obtain the equivalent inequality,


k-1

(5)

u. < n <

u.

so that by the Zeckendorf t h e o r e m , the non-negative i n t e g e r


k

1 u.I - n
1
p o s s e s s e s a n expan sion in the : form,
00

2 u.I - n = 1 a.u.

(6)

with coefficients satisfying


r

&

a. a. .. = 0

for

i >. 1.

l+l

Note from (5) that


k

2 u.I -
l

k
n

<

k-1

1 u.l -

u.1

'

A NEW CHARACTERIZATION OF

which i m p l i e s

a. - 0 for

February

i >. k i n ( 6 ) . I n p a r t i c u l a r ,

= 0o

Hence,

(6) m a y be r e w r i t t e n

V
^

u. - n = V, a.u.

k
n = X (1 - a.)u.
1

(?)

with

l i

w h e r e we have defined

k
- 2 .u.
1

/3. = 1 - a.
l

for

a, = 0 ,
k

with k = 1

i = 1, 2, . . , k.

It is c l e a r

from the r e l a t i o n s a .i a .l+l


and a .I a l+l
. , , ==(1
jS.,,^1
v -/ fl.)(l-/3.,
xl=0
r * M i +11 ' ) thatM|8.
i +M i+1
for i = 1, 2, o o . , k - 2, as required,,
To show u n i q u e n e s s ,

a s s u m e t h e r e e x i s t s a positive i n t e g e r

with two r e p r e s e n t a t i o n s :

(8)

n = 2

/S.u. = 2

jS.'u.

where ^
= 0 = 1, / ^ + i + 1 ^ * f o r
ySj' + / 3 . + 1 ' 2 1 for i = 1, 2, . . . . p - 2.
If m / p, t h e n w e a s s u m e

i =. 1. 2, . . . , m - 2; and

m > p without loss of g e n e r a l i t y , and

from the coefficient c o n s t r a i n t s and L e m m a 1, we have

jQ.u.
M

~ = u
- 1 ,
i i > u m +um - 20 +u m - 4. + . . . + u, 1,2
m+1

while
p
p
f
V fi. u. < V
**

u. <
i

m-1
V u. = u
^

,,

m+1

(Here and in what follows, the s u b s c r i p t notation u,

s e r v e s to indi-

cate the final t e r m in a sum, the value of the final t e r m being e i t h e r


u, or u ? depending on the p a r i t y of the index a s s o c i a t e d with the initial

1965

THE FIBONACCI NUMBERS

term in the sum. )

This is in evident contradiction to (8) and we con-

clude m = p.
Now, define
Then

a{ai+l

a = 1 - ]8. and a.' = 1 -.

for

i = 1, 2, . . . , p.

= a.
- a a:
^ a j ^ ==0
for i i == 1
0 for
.,1, 2,2, . .... ,, / pp -- 11, and (8) becomes
i i.+l
P
P
(l-a.'Ju.
(1 - a.)u. = 1
i

or
P
(9)

a.u.

= 2

a.'u.

Since both sides of (9) are admissible Zeckendorf representations, the


uniqueness of such representations implies

a. = a.'

for

i = l, 2, . . . , p

or equivalently fi. - /3.' for i = 1, 2, . . . , p, which proves uniqueness


of the dual representation and completes the proof of Theorem 1.
3.
Next,

THE CONVERSE THEOREM

we will show that the expansion property expressed in

Theorem 1 actually provides a characterization of the Fibonacci numbers,


Definition 2:

An arbitrary

sequence of positive integers,

{v;}>

i = 1, 2, o.. will be said to possess the dual unique representation


property (Property D) if and only if every positive integer

n has a

unique representation in the form


P
"(10)
\

n = X

fi.v.

with

' i i

B = 1,

and

'p

1
(11)
Corollary 1:

. + (3i+l

21

A sequence

elements; that is, v


4
m '

lv.\
f

for

i = 1,2, . . . . p - 2

possessing Property D has distinct


m

4 n

A NEW CHARACTERIZATION OF

February

Proof:

A s s u m e m 4 n and v = v . Take m > n > 1 without loss


'
m
n
of generality; then,
m
m -1

2 v t =. 2
i=l .
l

v. ,

contradicting the a s s u m e d uniqueness of expansions satisfying (10) and


(11).
L e m m a 2: If ( v. \ p o s s e s s e s P r o p e r t y D, then v = u,
w h e r e / u. \ is the Fibonacci sequence of Definition 1,
Proof:

In o r d e r to r e p r e s e n t the i n t e g e r 1 in the p r o p e r form

( 1 1 ) ] , it is c l e a r that e i t h e r
then v

and v? = u~,

v,

or v " m u s t be equal to 1.

- Z n e c e s s a r i l y and the L e m m a is p r o v e d .

If v = 1,

In the r e m a i n i n g

case,

v, = 2 , v n = 1 and it follows that v 0 = 3 and v = 6.


1
2
3
4
point, it is i m p o s s i b l e to r e p r e s e n t the i n t e g e r 8 in p r o p e r
m a t t e r how the r e m a i n i n g (distinct) v. a r e chosen. Thus v,
and v ? = 2 = u? a s s t a t e d .
T h e o r e m 20
If j v . \ , i = 1, 2, . . . is an a r b i t r a r y sequence
tive i n t e g e r s p o s s e s s i n g P r o p e r t y D, then v. = u. for all i k
Proof:

[(10)-

At this
form no
= 1 = u,
of p o s i 1.

The a s s e r t i o n is t r u e for i = 1 and 1 = 2 a s p r o v e d in L e m m a

20 Now, a s s u m e a s an induction hypothesis that v.. = u, , v? = u ? , ,


v, = u, for some k >. 2 We wish to show that v, , = u, . n e c e s s a r i l y . .
R e c a l l from T h e o r e m 1 (noting v. = u. for i = 1, 2, . . . , k by
the inductive a s s u m p t i o n ) that e v e r y positive i n t e g e r

0 < n <

v.
l

has a representation

n. = I

/Jv.

I I

satisfying

1965

THE FIBONACCI NUMBERS

wher e

m < k,

/S

= 1 and

We show first that

/3. + i + 1 > 1 for

v, , , > u, . , .
k+1
k+1

1 = 1 , 2, . . . ,

- 2.

For, if not, then v. M <u, M


k+1
k+1

and
(12)

k+l+Vk-l+Vk-3+- + V 1 , 2 < u k + l + u k - l + U k - 3 + ' + u l , Z^k+Z'1'

which implies

k+l*Vl+"-+Vl,2

^Uk+2~2

1
F r o m (12) and the r e m a r k in the preceding paragraph, we have
m
V

k+l+Vk-l+Vk-3+--+Vl,2

~~ 2

^i V i

'

1
with m < k,

R
= 1 and /?. +M# . , , >- 1 for i = 1, 2, ... . , m - 2.
H
m
*i
i+1
Since both sides a r e in the proper form and a r e not identical, uniqueness

is contradicted.

Therefore

v, . , > u. . . as a s s e r t e d .
k+1
k+1
Now a s s u m e v, , > u, , We shall show that this assumption
also leads to a contradiction of uniqueness. If v, , , > u, , , then the

unique representation of the integer


k

2
has the form

k
(13)

u. + 1

k+m
u.+l

0-v.

with m > 2, ^

= 1 and 0 ^ / 3 ^ > 1

1
for

i = 1, 2, . . o , k+m-2.

(For, if m < 2 in (13), then m = 1 since


with non-vanishing coefficient.
k+1
2

k + m

/3.v.
H
i i

v,+1

must certainly appear

But

> v, , , +u, , -hi, +. . . +u, 9 > u, , , +u, , +u, ~+0 o . -hi, ? =


k+1 k-1 k-3
1,2
k+1 k-1 k-o
1,6

k
u.+l, so that (13) could not be valid.)
1
The foregoing argument also shows J3I__LI = 0 in (13); hence
=

j3i = /9i , ? = 1 from the coefficient constraints, and

A NEW CHARACTERIZATION OF

F e b r u a : try

k+m

ft

*i

v.

A{

u.+l

k+2+Uk+Uk-2+-

+U

T,2

1
k-I
V

k+2+Uk+l "

II

k+2

+ 1

],

= u k>

1
or

k+2-

From

C o r o l l a r y 1, w e i n f e r ( n o t e

(14)

vk+2 < uk

Now, c o n s i d e r t h e p a r t i c u l a r i n t e g e r ,

k+2+Vk+Vk-2+- ' '

+V

1, 2 <

that

N=v,

~ + v

w h i c h i s i n t h i s a d m i s s i b l e f o r m of (10) - ( 1 1 ) .
(15)

k+(VUk-2+'

+ v,

+ . . . + v,

We h a v e

+ u

= u

k+uk+l"1 ^k-KT

or

?i

l,2

k
N

+V +V

k+2

+D . . +V

k-2

-2 = 2

< U

1,2-

k+2

v.
I

Thus N also h a s a r e p r e s e n t a t i o n in a d m i s s i b l e form involving at m o s t


the first

k m e m b e r s of t h e s e q u e n c e

{ v . \ , and uniqueness is c o n t r a -

dicted.
T h e i n e q u a l i t yJ v, ,, > u, , , , i s t h e r e f o r e u n t e n a b l e a n d w e h a v e
^
k+1
k+1
s h o w n v, , i = u, , , T h e t h e o r e m t h e n f o l l o w s i m m e d i a t e l y b y i n d u c t i o n .
T h u s , t h e d u a l u n i q u e r e p r e s e n t a t i o n p r o p e r t y ( P r o p e r t y D) i s a
p r o p e r t y enjoyed only by the F i b o n a c c i n u m b e r s and is t h e r e f o r e

suf-

ficient to c h a r a c t e r i z e the s e q u e n c e / u . \ .
Acknowledgement:
fessor

I w o u l d l i k e t o a c k n o w l e d g e t h e c o n t r i b u t i o n of P r o -

V e r n e r E . Hoggatt, J r . ,

t h e o r e m s of t h i s p a p e r .

w h o s e c a t a l y t i c c o m m e n t s l e d t o the

S e e a l s o a p a p e r b y H . H . F e r n s [3] t h i s i s s u e .

REFERENCES
1.

J . L. B r o w n , J r . , " Z e c k e n d o r f f s T h e o r e m a n d S o m e A p p l i c a t i o n s , " T h e F i b o n a c c i Q u a r t e r l y , V o l . 2 , N o . 3, p p . 1 6 3 - 1 6 8 .

2.

D . E . D a y k i n , " R e p r e s e n t a t i o n of N a t u r a l N u m b e r s a s S u m s of
G e n e r a l i z e d F i b o n a c c i N u m b e r s , " J o u r n a l of t h e L o n d o n M a t h e m a t i c a l Society, 35(1960), pp. 43-160.
H H . F e r n s , " O n t h e R e p r e s e n t a t i o n of I n t e g e r s a s S u m s of D i s t i n c t F i b o n a c c i N u m b e r s , " T h e F i b o n a c c i Q u a r t e r l y , V o l . 3, N o .

3.

1, p p . 2 1 - 2 9

XXXXXXXXXXXXXXX

SOME RESULTS CONCERNING POLYOMfNOES


DAVID A. KLARNER
University of Alberta, Edmonton, Alberta

INTRODUCTORY REMARKS
An n - o m i n o is a plane figure composed of
s q u a r e s joined edge on edge.

connected unit

In the e a r l y nineteen h u n d r e d s , H e n r y

Dudeney, the famous B r i t i s h puzzle expert, and the F a i r y Chess Review


p o p u l a r i z e d p r o b l e m s involving n - o m i n o e s which they r e p r e s e n t e d a s
figures cut from c h e c k e r b o a r d s ,

Solomon Golomb s e e m s to have been

the f i r s t m a t h e m a t i c i a n to t r e a t the subject s e r i o u s l y when as a g r a d uate student at H a r v a r d in 1954,

he published " C h e c k e r b o a r d s and

P o l y o m i n o e s " in the A m e r i c a n M a t h e m a t i c a l Monthly.

Since 1954,

s e v e r a l a r t i c l e s have a p p e a r e d (see R e f e r e n c e s ) ; in p a r t i c u l a r , R C
Read [9] and M u r r a y Eden [2] have d i s c u s s e d the p r o b l e m of finding or
e s t i m a t i n g the n u m b e r

p(n) of n - o m i n o e s for a given n.

r e s u l t s it is now known that for l a r g e


n
cx
where

c,

and c ?

, x
< p(n)

From their

n
n
< c2

a r e c e r t a i n positive constants g r e a t e r than 1.

In

the f i r s t p a r t of this p a p e r we e n u m e r a t e a subset of n - o m i n o e s and


provide an i m p r o v e d lower

bound for

p(n); l a t e r we d i s c u s s other

p r o b l e m s of this s o r t and conclude with a brief exposition of p r o b l e m s


dealing with configurations of n - o m i n o e s .
MOSER'S BOARD P I L E P R O B L E M
In the following it will be convenient to have c e r t a i n conventions,,
We say the region between y = n-1 and y = n is the n
a r e c t a n g l e of width one a s t r i p .

row and call

The f i r s t s q u a r e on the left in a s t r i p

located in a row is called the initial s q u a r e of the s t r i p ; an n - o m i n o is


located in the plane when s o m e s q u a r e in the n - o m i n o exactly c o v e r s
a s q u a r e in the plane l a t t i c e . The set of all incongruent n - o m i n o e s will
be denoted by P(n) and for convenience we think of the e l e m e n t s of
P(n) located in a r b i t r a r y r e g i o n s of the plane.
position due to t r a n s l a t i o n s ,

Ignoring changes in

each e l e m e n t of P(n) h a s eight or l e s s


9

10

SOME RESULTS CONCERNING POLYOMINOES

p o s i t i o n s w i t h r e s p e c t t o 90

February

rotations about the origin and reflections

a l o n g t h e x o r y a x e s ; t a k i n g t w o n - o m i n o e s t o be d i s t i n c t if o n e c a n n o t
be t r a n s l a t e d t o c o v e r t h e o t h e r ,

w e find a n e w s e t

by i n c l u d i n g r o t a t i o n s a n d r e f l e c t i o n s

S(n)

of n - o m i n o e s i n

from

P(n)

P(n)

in

S(n).

T h e p r o b l e m w h i c h i s n o w t o be d i s c u s s e d w a s p r o b a b l y f i r s t
p o s e d by L e o M o s e r i n p r i v a t e c o r r e s p o n d e n c e w i t h t h e p r e s e n t a u t h o r ;
l a t e r h e p o s e d it in a d i f f e r e n t

f o r m a t t h e 1963 N u m b e r T h e o r y C o n -

f e r e n c e h e l d a t t h e U n i v e r s i t y of C o l o r a d o .
the p r o b l e m ,

but h i s r e s u l t s

The p r o b l e m

is to e n u m e r a t e a s u b s e t

E d e n [2] a l s o d i s c u s s e s

a r e not as c o m p l e t e as those given h e r e .


B(n)

of

S(n)

which

contains

n - o m i n o e s h a v i n g t h e p r o p e r t y t h a t t h e y c a n be t r a n s l a t e d i n s u c h a w a y
that t h e y a r e e n t i r e l y in the f i r s t and s e c o n d q u a d r a n t s with e x a c t l y
one s t r i p in the f i r s t r o w with its i n i t i a l s q u a r e at the o r i g i n and e a c h
r o w a f t e r t h e f i r s t h a s no m o r e t h a n o n e s t r i p i n i t .

Such n - o m i n o e s

m a y b e v i s u a l i z e d a s s i d e e l e v a t i o n s of b o a r d p i l e s c o n s i s t i n g of b o a r d s
of v a r i o u s

lengths which g e n e r a l l y h a v e not been s t a c k e d

carefully,

s e e F i g u r e 1.
M o s e r n o t e d t h a t if

b(n)

d e n o t e s t h e n u m b e r of e l e m e n t s i n

B(n),

then

(1)

b(n)

= X ( a 1 + a 2 - l ) ( a 2 + a 3 - 1) . . . ( a . ^ + a. - 1)

w h e r e t h e s u m m a t i o n e x t e n d s o v e r a l l c o m p o s i t i o n s a, + a ? . 0 . + a. = n
of n .
T h e r e l a t i o n i n (1) c a n be e s t a b l i s h e d b y t h e f o l l o w i n g c o m b i natorial argument.
F o r e a c h c o m p o s i t i o n a , + a~ + , . . + a. of n

1
Z
I
t h e r e i s a s u b s e t of B(n) c o n s i s t i n g of n - o m i n o e s w h i c h h a v e a s t r i p
of

s q u a r e s in the

o m i n o e s i n e a c h of t h e s e

row

(t = 1, 2, . . . , i); t h e n u m b e r

s u b s e t s i s 1 if

of n -

i = 1 w h i c h c o r r e s p o n d s to

t h e v a l u e of t h e e m p t y p r o d u c t i n t h e s u m (in t h i s t h e r e i s a s t r i p
u n i t s long in the f i r s t

row) and

(a, + a~ - 1) ( a ? + a~ - 1) . . .

(a. , +

+ a . - 1) if i > 2 T h i s f o l l o w s s i n c e t h e r e a r e e x a c t l yJ (a^ , + a , - 1)
l
.,
t-1
t
w a y s to j o i n t h e s t r i p of a
s q u a r e s in the t
r o w to t h e s t r i p of
a

s q u a r e s i n t h e r o w b e l o w a n d t h e t o t a l n u m b e r of w a y s t o c o n n e c t

u p t h e s t r i p s t o f o r m a n n - o m i n o w o u l d be t h e p r o d u c t of a l l of t h e s e
alternatives,,

T h e s u b s e t s c o r r e s p o n d i n g to t h e c o m p o s i t i o n s of

are

1965

SOME RESULTS CONCERNING POLYOMINOES

11

exhaustive and disjoint in B(n), so that b(n) is the sum of the n u m b e r


of e l e m e n t s in each subset s which is (1).
The r e l a t i o n for

b(n) given by (1) does not furnish a v e r y handy

device for computing


mating

b(n).

b(n), but as Eden h a s shown it is helpful in e s t i -

Rather

than a t t e m p t to sum

(1) by p u r e l y a l g e b r a i c

m a n i p u l a t i o n s , we r e t a i n the g e o m e t r i c i n t e r p r e t a t i o n of the p r o b l e m
so that c o m b i n a t o r i a l a r g u m e n t s can be m o r e e a s i l y applied toward
finding a r e c u r s i o n r e l a t i o n for

b(n)Q

To find a r e c u r s i o n r e l a t i o n for
(r = 1, 2, . . . , n)
exactly r

of

B(n)

b(n) we define s u b s e t s

B (n)

which contain n - o m i n o e s with a s t r i p of

s q u a r e s in the f i r s t row and let b (n) denote the n u m b e r

of e l e m e n t s in B (n).

It is obvious that the s u b s e t s

B (n) (r = 1, 2,

. . . , n) a r e exhaustive and disjoint in B(n) s o t h a t w e have i m m e d i a t e l y


n
(2)

b(n) -

X b r (n)
r=l

Bv definition of

B (n), b (n) = 1.
Consider the e l e m e n t s of
n
n
B (n) with r < n; each e l e m e n t of B (n) c o n s i s t s of a s t r i p of r
r
r
s q u a r e s in the f i r s t row with some e l e m e n t of B(n-r) located in the
rows above the f i r s t . The situation can be a p p r a i s e d m o r e c o n c i s e l y
J

when one c o n s i d e r s the n u m b e r of ways an e l e m e n t of the subset


of B(n-r)

can be attached to the s t r i p of r

B.(n-r)

s q u a r e s in the f i r s t row

so that the n - o m i n o e s formed will be an e l e m e n t of B (n).

Clearly

this can be done in r + i - 1 ways, so that exactly (r + i - 1) b. ( n - r )


of the e l e m e n t s of B (n) have an element of B.(n-r)
r

s t r i p of

s q u a r e s in the f i r s t row.

(i = 1, 2, . . . , n - r )

of

B(n-r)

connected to the

Since the s u b s e t s

a r e exhaustive,

B-(n-r)

disjoint s u b s e t s ,

it

follows that
n- r
(3)

b (n) =

(r + i - 1) b . ( n - r )

for

r < n

i=l
It will be s e e n p r e s e n t l y that the r e l a t i o n s in (2) and (3) a r e
enough to find the d e s i r e d r e c u r s i o n r e l a t i o n for

b(n).

Before this

12

SOME RESULTS CONCERNING POLYOMINOES

February

r e s u l t can be given, we have to p r o v e a few l e m m a s .


L e m m a 1:
Proof:

If n > 1, b (n) - b T (n-1) = b ( n - r ) 0


r-1
r*
Using (3) it is s e e n that
n-r

b r (n) - b r _ x ( n - l )

n-r
(r+i-l)b.(n-r)

i=l

(r+i-2)b.(n-r)

i=l

n-r
=

b.(n-r)

i=l
but a c c o r d i n g to (2), the l a s t e x p r e s s i o n is p r e c i s e l y

b ( n - r ) , so the

proof is finishedo
L e m m a 2:
Proof:
that

If n > 1, b(n) = 2 b ( n - l ) + b (n) - b ^ n - 1 ) .

Using r e l a t i o n s for b(n) and b ( n - l )

n
(5)

b(n) - b ( n - l ) =

given by (2), it is s e e n

n-1
b (n) -

i=l

b.(n-l)

i=l.
n-1

= b x (n) +

2 jb.(n) - b . ^ n - l ) !
i=2

but a c c o r d i n g to L e m m a 1, b(n-i) can be substituted for b.(n) - b.

(n-1)

i n the l a s t m e m b e r of (5) so that making this substitution and t r a n s posing

- b ( n - l ) from the f i r s t to the l a s t m e m b e r gives


n-1

(6)

b(n) = b x (n) +

b(n-i)

i=l
Now using r e l a t i o n s given by (6) for b(n) and b ( n - l ) we have
n-1
(7) b(n) - b ( n - l )

= b^n) +

n-2
b(n-i) - b ^ n - 1 )

i=l
= b^n) - b,(n-l) + b(n-l);

2
i=l

b(n-l-i)

1965

SOME RESULTS CONCERNING POLYOMINOES

13

the d e s i r e d r e s u l t is obtained by adding b ( n - l ) to the f i r s t and last


m e m b e r s of (7).
L e m m a 3:
Proof:

t^ (n) = 4 b ^ n - 1 ) - 4 b ^ n - 2 ) + b ^ n - 3 ) + 2 b(n-3).

Taking

r = 1 in (3) gives an e x p r e s s i o n for

b, (n); namely,

n-1
(8)

b x (n) =

ib.(n-l)

i=l
Using r e l a t i o n s for
stituting b(n-2-i) for

b, (n)

and

b, (n-1)

given by (8) and s u b -

b . + , ( n - 1 ) - b.(n-2) and b ( n - l )

for

n-1
2

b.(n-l)

i=l
when they o c c u r , it is s e e n that
n-1
(9) b x (n) - b ^ n - 1 ) =

n-2
i b.(n-l) -

i=l

i b.(n-2)

i=l

n-1
=

n-2
b (n-1) + 1

i=l

n-2
ib.+1(n-l)-

i=l

i b.(n-2)

i=l

n-2
= b(n-l) + 2

i ) b . + 1 ( n - l ) - b.(n-2)j

i=l
n-2
= b(n-l) + 2

i b(n-2-i)

i=l
Adding b, (n-1) to each m e m b e r of the equality and dropping the
l a s t t e r m in the sum in the right m e m b e r of (9) (since b(0) - 0) a new
r e l a t i o n for

b, (n) is obtained:
n-3

(10)

b^n)

= b ^ n - 1 ) + b(n-l)

2
i=l.

(n-2-i) b(i)

14

SOME RESULTS CONCERNING POLYOMINOES


This t i m e using e x p r e s s i o n s for

and again writing a r e l a t i o n for

b, (n) and b , ( n - l )

February
givenby(lO)

b, (n) - b (n-1), one obtains after a

few a l g e b r a i c manipulations
n-1
(11)

b^n)

= 2^(11-1) - b (n-2) - 2b(n-2)

b(i)

i=l
Repeating the s a m e p r o c e d u r e as before only this t i m e using e x p r e s s i o n s for
(12)

b, (n) and b ^ n - l )

given by (11) yields

b x (n) = S b ^ n - 1 ) - 3b x (n-2) + ^ (n-3) + b(n-1) - 2b(n-2) + 2b(n-3);

but by L e m m a 2, b ( n - l ) - 2b(n-2) = b , ( n - l ) - b, (n-2)

so that s u b s t i -

tuting the l a t t e r quantity for the f o r m e r in (1 2) gives the d e s i r e d r e s u l t .


T h e o r e m 1:

Proof:

b(l) = 1, b(2) = 2, b(3) = 6, b(4) = 19, and


b(n) = 5 b ( n - l ) - 7b(n-2) + 4b(n-3) for n > 4.

The values of b(i) (i = 1,.2,

3, 4) can be computed d i r e c t l y

from (1) or by taking b(l) = b, (1) = 1 the r e l a t i o n s in (2) and (3) can
be used together for the s a m e p u r p o s e .
l i n e a r difference equations involving

L e m m a s 2 and 3 provide the


b, (n)

and

b(n)

which can be

u s e d to find
(13)
(14)

b(n) = 5 b ( n - l ) - 7b(n-2) + 4b(n-3) ,


b x (n) = 6b (n-1) - 12b (n-2) + l l b ( n - 3 ) - 4b(n-4),

which c o m p l e t e s the proof.


The a u x i l i a r y equation for (13) has one r e a l root g r e a t e r than 3 2
so that for

n sufficiently l a r g e
b(n) > ( 3 . 2 ) n .

(15)

We conclude from e a r l i e r r e m a r k s that


b(n)/8 incongruent n - o m i n o e s ,

B(n) contains at least

so that we can a l s o r e p l a c e

b(n) in

(15) with p(n).


Having disposed of the m o r e difficult p r o b l e m f i r s t , we now t u r n
attention to solving an e a s i e r and r e l a t e d p r o b l e m which was posed and
solved by M o s e r .

1965

SOME RESULTS CONCERNING POLYOMINOES

15

Let C(n) be the subset of B(n) which contains all n - o m i n o e s


having the p r o p e r t y that the initial s q u a r e of the s t r i p in the k

row
is no further to the left than the initial s q u a r e of the s t r i p in the (k-1) st
row.

R e c a l l from the definition of B(n) that the initial s q u a r e of the

s t r i p in the f i r s t row is always located at the origin.

Using a c o m b i -

n a t o r i a l a r g u m e n t s i m i l a r to the one provided for the proof of (1), it is


e a s y to prove
(16)

c(n) =

a. a 2 . . . a. ^

a, +a~+. ... +a.=n


1 Z
i
where

c(n) denotes the n u m b e r of e l e m e n t s in C(n).

Applying the

m e t h o d s he gave in [8] , M o s e r was able to show from (16):


T h e o r e m 2:

c(n) is equal to the

(2n-l)

st

Fibonacci n u m b e r .

We will give an a l t e r n a t e proof using the s a m e idea used in the


proof of T h e o r e m 1. Let C.(n)

be the subset of C(n) which contains

all n - o m i n o e s having s t r i p s of exactly


C l e a r l y the s u b s e t s

s q u a r e s in the f i r s t row.

C.(n) (i = 1, 2, . . , n) a r e exhaustive and d i s -

joint in C(n) so that letting c.(n) denote the n u m b e r of e l e m e n t s in


C.(n) we have
i

n
(17)

c(n) =

2
i=l

c.(n)

Next, it is e a s y to see that c (n) = 1, and for i < n, c.(n) = i c(n-i)


since e a c h element.of

C(n-i) can be joined exactly i ways to the s t r i p

of i s q u a r e s in the f i r s t row so as to form an e l e m e n t of C.(n); the


n - o m i n o e s thus formed obviously c o m p r i s e all the e l e m e n t s of
Substituting the e x p r e s s i o n s just found for

C.(n).

c.(n) into (17) we obtain

n-1
(18)

c(n) = 1 + 2

i c(n-i)

i=l
Using e x p r e s s i o n s for c(n) and c ( n - l )
bine the s u m s in c(n) - c ( n - l )

to find

given by (1 8) we can c o m -

16

SOME RESULTS CONCERNING POLYOMINOES

February

n-1
c(n) - c ( n - l ) =

c(i) ,

i=l
or
n-1
c(n) = c ( n - l ) + '

c(i)

i=l
Now using e x p r e s s i o n s for c(n) and c ( n - l )
combine the s u m s in c(n) - c ( n - l ) and deduce
(20)

given by (19) we can

c(n) = 3 c ( n - l ) - c(n-2).
It is e a s y to p r o v e that the Fibonacci n u m b e r s with odd indices

satisfy the r e c u r r e n c e r e l a t i o n i n (20). Also, using (16) we find c(l) = f,


and c(2) = f~

(f. denotes the i

the s e q u e n c e s

Fibonacci n u m b e r as usual) so that

{c.} and {f ? . ,} m u s t be identical. E d i t o r i a l Note:

See H-50 Dec. 1964 and note notational d i f f e r e n c e s .


N-OMINOES ENCLOSED IN RECTANGLES
R0 C. Read [9] h a s t r e a t e d the p r o b l e m of e n u m e r a t i n g the nominoes which "fit" into a p x q r e c t a n g l e . An n - o m i n o is said to fit
in a p x q r e c t a n g l e if it is the s m a l l e s t r e c t a n g l e in which the n - o m i n o
can be d r a w n with the s i d e s of its s q u a r e s p a r a l l e l to the sides of the
r e c t a n g l e . R e a d ' s m e t h o d s give exact counts of the n - o m i n o e s in the
s e t s c o n s i d e r e d ; however, it is p o s s i b l e to obtain lower bounds for t h e s e
n u m b e r s with l e s s effort using s i m i l a r i d e a s . To i l l u s t r a t e we will
c o n s i d e r the p r o b l e m of e s t i m a t i n g from below the n u m b e r s ? (n) of
n - o m i n o e s which fit in a 2 x k r e c t a n g l e ; we call this set of n - o m i n o e s
S ? (n). Two e l e m e n t s a r e distinct if they a r e incongruent, so S~(n) is
a subset of P(n).
F i r s t , we o b s e r v e that each e l e m e n t of S ? (n) can be located ent i r e l y in the f i r s t q u a d r a n t in rows 1 and 2 with a s q u a r e Located at the
origin. If each e l e m e n t of S ? (n) is situated in the way j u s t d e s c r i b e d
in e v e r y w a y p o s s i b l e , a new set U(n) is obtained w h e r e two e l e m e n t s
a r e distinct if one does not exactly cover the other 0 C l e a r l y , u(n), the

1965

SOME RESULTS CONCERNING POLYOMINOES

17

n u m b e r of e l e m e n t s in U(n), is l e s s than or equal to 4 s ? ( n ) .


U(n) can be divided into two s e t s

U"(n) and

Now

U'(n) consisting r e s p e c -

tively of n - o m i n o e s having and not having a s q u a r e in the second row


attached to the s q u a r e at the origin.

Let the n u m b e r of e l e m e n t s in

U'(n) and U"(n) be u'(n) and u"(n)

respectively.

Now it is e a s y to

see that
u f {n) = u ' ( n - l ) + u " ( n - l )

(21)

since e v e r y e l e m e n t of U"(n-1)

and U ' ( n - l )

can be t r a n s l a t e d a unit

to the right of the origin and a s q u a r e located at the origin to give an


e l e m e n t of U'(n) and e v e r y e l e m e n t is obviously obtained in this fashion.

It is a l s o e a s y to p r o v e

(22)

u"(n) = 2u'(n-2) + u"(n-2)

since e v e r y e l e m e n t of U"(n-2) and e v e r y e l e m e n t of U'(n~2) and its


h o r i z o n t a l reflection can be t r a n s l a t e d a unit to the right of the origin
and two s q u a r e s added (one at the origin,

the other attached above it)

to form e v e r y e l e m e n t of U"(n).
Using (21) and (22) we can find
(23)

u'(n)

= u'(n-l) +u'(n-2)

+u'(n-3)

and

uM(n) = u " ( n - l ) + u"(n-2) + u M (n-3),

(24)

so that it b e c o m e s evident from


(25)

u(n) = u ( n - l ) + u(n-2) + u(n-3)


Since

s?(n).

u(n) = u'(n) + u"(n)

u(n)/4 < s ? (n) s

(25) p r o v i d e s

that
.

a r e l a t i o n for

estimating

The s a m e p r o c e d u r e can be used for e s t i m a t i n g the n u m b e r of

e l e m e n t s in S,(n) consisting of n - o m i n o e s which fit in k x q r e c t a n g l e s .


N-OMINO CONFIGURATIONS
P r o b l e m s involving n - o m i n o configurations have enjoyed a g r e a t
p o p u l a r i t y among m a t h e m a t i c a l r e c r e a t i o n i s t s [4] , [6] .

We plan to

devote a s m a l l amount of space to giving an exposition of p r o b l e m s


which m a y be of i n t e r e s t to the m a t h e m a t i c i a n . G e n e r a l l y t h e s e p r o b l e m s

18

SOME RESULTS CONCERNING POLYOMINOES

have the following form:

February

given a region of a r e a A and a set of n-

ominoes having a combined a r e a a l s o A; can one cover the region with


the s e t ?
We say a set exactly c o v e r s a region when t h e r e is no o v e r l a p
and no p a r t of the region is left u n c o v e r e d .

It would be i n t e r e s t i n g to

know n e c e s s a r y conditions that an n - o m i n o be such that an unlimited


n u m b e r of copies could be used to exactly cover the plane 0

A related

p r o b l e m is to d e t e r m i n e n e c e s s a r y conditions that some n u m b e r of


copies of a given n - o m i n o could be used to exactly cover a r e c t a n g l e .
Thus, some e a s i l y proved n e c e s s a r y conditions a r e given by:
(i)

if an n - o m i n o has two lines of s y m m e t r y and a set of t h e s e nominoes exactly c o v e r s a r e c t a n g l e , then the n - o m i n o is itself a
rectangle.

(ii)

if an n - o m i n o fits in a p x q r e c t a n g l e and c o v e r s diagonally opposite c o r n e r s of the r e c t a n g l e , and a set of t h e s e n - o m i n o e s can


be used to exactly cover a r e c t a n g l e ,

then the n - o m i n o is itself

a rectangle.
A r e c t a n g l e exactly c o v e r e d with a set of congruent n - o m i n o e s is
m i n i m a l when no r e c t a n g l e of s m a l l e r a r e a can be exactly c o v e r e d with
a set of the s a m e n - o m i n o e s containing fewer e l e m e n t s .

It is e a s y to

p r o v e that t h e r e is an unlimited n u m b e r of m i n i m a l r e c t a n g l e s involving


e i t h e r two or four n - o m i n o e s .

F i g u r e s 2, 3, 4 and 5 show i n s t a n c e s of

m i n i m a l r e c t a n g l e s involving m o r e than four n - o m i n o e s .

Are there

infinitely m a n y c a s e s of m i n i m a l r e c t a n g l e s which involve m o r e than


four n - o m i n o e s (no two c a s e s involving s i m i l a r n - o m i n o e s ) ? A r e t h e r e
m i n i m a l r e c t a n g l e s involving an odd n u m b e r of n - o m i n o e s which a r e not
themselves rectangles?
Note that the configurations depicted in F i g u r e s 1, 2, 3 and 4 a r e
s y m m e t r i c with r e s p e c t to the c e n t e r s of the r e c t a n g l e s .

Can this a l -

ways be done in m i n i m a l r e c t a n g l e s ?
GENERALIZATIONS OF N-OMINOES
In [5] , Golomb s u g g e s t s that one could t r y to d e t e r m i n e or e s t i m a t e the n u m b e r of distinct ways n e q u i l a t e r a l t r i a n g l e s or n r e g u l a r

1965

SOME RESULTS CONCERNING POLYOMINOES

hexagons could be simply connected edge on edge.

19

Using 1, 2, 3, 4,

5 or 6 hexagons 1, 1, 3, 7, 22 or 83 combinations respectively result;


so far no upper or lower bounds for the terms of this sequence have
been given.
There is no reason why regular k-gons could not be used for
cells in such combinatorial problems; overlapping of cells could be
permitted so long as no cell exactly covered another.

Thus, where

at most four squares or three hexagons might have a vertex in common, at most ten pentagons might have a vertex in common.

The num-

ber of distinct ways to join two regular k-gons is one; the number of
ways to join three regular k-gons is the greatest integer in k/2.

Per-

haps it would not be difficult to determine in how many ways four or


five regular k-gons could be joined together edge on edge so that distinct simply connected figures are formed.
Still another generalization of n-ominoes which seems not to
have been considered is joining squares together edge on edge in three
or more dimensions.

The number of ways of joining k cubes face on

face in three dimensions (including mirror images of some pieces) is


1, 1, 2, 8, 29j and 166 for

k = 1, 2, 3, 4, 5, and 6 respectively; no

bounds have been given for the terms of this sequence nor has much
been done in a serious vein connected with the packing of space with
these three dimensional analogues of polyominoesD

REFERENCES
1.

T. R. Dawson and W. E. Lester,

"A Notation for Dissection

Problems, " The Fairy Chess Review, Vol. 3, No. 5 (April 1957),
pp 46-47.
2.

M. Eden, "A Two-Dimensional Growth Process, "Proceedings of


the Fourth Berkeley Symposium on Mathematical Statistics and
Probability, Berkeley and Los Angeles, University of California
Press, (1961), pp0 223-239.

3.

M. Gardner, The 2nd Scientific American Book of Mathematical


Puzzles and Diversions, New York, Simon and Schuster, (1961),
pp. 65-880

SOME RESULTS CONCERNING POLYOMINOES

February

Mo G a r d n e r , " M a t h e m a t i c a l G a m e s , " Scientific A m e r i c a n , Vol.


196,

( D e c , 1957), pp e 126-134.

S. Wo Golomb,

" C h e c k e r b o a r d s and P o l y o m i n o e s , " A m e r i c a n

M a t h e m a t i c a l Monthly, Vol 61, (1954), pp. 675-82.


, "The G e n e r a l T h e o r y of P o l y o m i n o e s , " R e c r e a t i o n a l
M a t h e m a t i c s Magazine, No. 's 4, 5, 6 and 8, (1962).
, "Covering a Rectangle with L - t e t r o m i n o e s , " A m e r ican M a t h e m a t i c a l Monthly, Vol. 70, (1961), pp 0 3 9 - 4 3 .
L. M o s e r a n d E o L. Whitney, "Weighted Compositions, " Canadian
M a t h e m a t i c a l Bulletin, Vol 0 4, No. 1, (1961), pp. 3 9 - 4 3 ,
R. C. Read,

"Contributions to the Cell Growth P r o b l e m , " Ca-

nadian J o u r n a l of M a t h e m a t i c s , Vol. 14, (1962), pp. 1-20.


W. Stead, "Dissection, " The F a i r y Chess Review, Vol. 9, No. 1,
(Dec.

1954), pp. 4 6 - 4 7 .

S Wo Golomb, " P o l y o m i n o e s , " C h a r l e s S c r i b n e r , N. Y., 1965 0

d
h L
1 r ~i1r
r
L ^ n

LL

F1
L
M

rn

r^

\-\

h
F
~n
xxxxxxxxxxxxxxx

ON THE REPRESENTATION OF INTEGERS AS SUMS


OF DISTINCT FIBONACCI NUMBERS
H.H. FERNS
University of Victoria, Victoria, British Columbia

This paper gives an e l e m e n t a r y d i s c u s s i o n of the problem of exp r e s s i n g an a r b i t r a r y positive i n t e g e r as a sum of distinct Fibonacci
numbers.

The r e c u r s i v e r e l a t i o n

(1)

4.7

.1

n+2
n+1
n
together with F , = F~ = 1 is used as the definition of Fibonacci n u m bers.

No use will be made of F ,

in any r e p r e s e n t a t i o n .

As an example c o n s i d e r the integer 29.

It m a y be e x p r e s s e d as

a sum of Fibonacci n u m b e r s in the following ways:


29v = F + F , = F + F +F = F + F + F +F
*8
6
8 5 4
8 5 3 2
= F + F , + F +F = F + F , + F +F +F
*7
6 5 4
7 6 '5 ^ 3
2
From

this example it i m m e d i a t e l y b e c o m e s a p p a r e n t that we

shall need to i m p o s e some "ground r u l e s " if we a r e to differentiate


between the v a r i o u s types of r e p r e s e n t a t i o n s .

This leads us to the

following definitions.
A r e p r e s e n t a t i o n will be called m i n i m a l if it contains no two consecutive Fibonacci n u m b e r s .
A r e p r e s e n t a t i o n is said to be m a x i m a l if no two consecutive
Fibonacci n u m b e r s
tion, w h e r e

F. and F . , ,
l

a r e omitted from the r e p r e s e n t a -

i+l

F~ ^L. F . < F. ,, 5L F
and F is the l a r tog e s t Fibonacci
2
I
i+l
n
n
n u m b e r involved in the r e p r e s e n t a t i o n .
Thus F ft + F/ is a m i n i m a l r e p r e s e n t a t i o n of the i n t e g e r 29
while

F ^ + F / +F,-+F 0 +F~ is a m a x i m a l r e p r e s e n t a t i o n .
7 6 5 3 2
It follows that a m a x i m a l (minimal) r e p r e s e n t a t i o n m a y be t r a n s -

formed into a m i n i m a l (maximal) one by the application or r e p e a t e d


application of (1).
We shall f i r s t r e s t r i c t our attention to m i n i m a l r e p r e s e n t a t i o n s .
As an i l l u s t r a t i v e example of m i n i m a l r e p r e s e n t a t i o n s we cons i d e r the r e p r e s e n t a t i o n s of all i n t e g e r s
21

N such that

ON THE REPRESENTATION OF INTEGERS AS

22

February

F? < N < F8
Thus

N m a y be any one of the eight i n t e g e r s 13, 14, 15, 16, 17,18,

19, or 20. Now the s m a l l e s t i n t e g e r in this set, n a m e l y 13, cannot be


r e p r e s e n t e d by the Fibonacci n u m b e r s

F ? , F~, F ,, F,- and F / ,

since

the l a r g e s t i n t e g e r under the m i n i m a l r e p r e s e n t a t i o n rule which they


can r e p r e s e n t is
F

6+F4+F2

12

Hence to r e p r e s e n t all i n t e g e r s of this set r e q u i r e s


FJ-J

F/

and

F ? , F~, F .,

F?.

By t r i a l we obtain the following r e p r e s e n t a t i o n s


13 = F ? ; 14 = F ? + F 2 ; 15 = F ? + F 3 ; 16 = F ? + F 4 ;
17 = F -7+ F4. +2F , ; 18 = F^+F^;
19 = F+F._+F
7 5
7 5 29 ;
20 = F ? + F 5 + F 3

One of t h e s e i n t e g e r s ,

n a m e l y 13, r e q u i r e s only one Fibonacci

n u m b e r to r e p r e s e n t it. F o u r of them, namely, 14, 15, 16 and 18 r e q u i r e two and t h r e e of them

17, 19, and 20 r e q u i r e t h r e e .

Let U
denote the n u m b e r of i n t e g e r s N in the range F ^.
to
to
n, m
n
N<F ,, which r e q u i r e m Fibonacci n u m b e r s to r e p r e s e n t t h e m .
Then
U7fl = l;U7>2 = 4;U7i3 = 3
It is also evident that
U

7, 1

+ U

7, 2

+ U

7, 3 = F8 -

Now it is known (1) that


U
n, m

= 0, if m > [ 2 ]
L J
2

Thus we m a y w r i t e
n

1
i=l

U . = F ,. - F = F .
n, i
n+1
n
n-1

1965

SUMS OF FIBONACCI NUMBERS


Table I gives values of U
for
to
n, m

2, 3, . . . ,

23

n = 1, 2, 3, . . . .

8 ; m = 1.
'

4.

We now d i s c u s s some p r o p e r t i e s of the function

n, m
Consider i n t e g e r s P, Q and R, defined by the following r e l a t i o n s
F < P < F ^ . ; F
< Q < F ; F ~ <R < F ,
n
n+1
n-1
n
n-2
n-1
Thus
(2)

P = Fn+Pf

(3)

n-1

(4)

p = 0 , 1 , 2 , ...-," F n _ r l
'H'

R-Fn_2+r,

P=Fn+Pl,

(B) P = F n + p 2 , pz=

"' " "

r = 0,1,2,...,

We a r r a n g e the i n t e g e r s
(A)

"

Pl

- 2

F ^ - l

P in two s e t s (A) and (B) a s follows.


= 0,1,2,...,

Fn_2-1

F n . 2 , F n _ 2 +l,Fn_2+2

Fn_2+(Fn_rFn_2-l)

= Fn_2+r, r = 0 , 1 . 2 , . . . ,

F ^ - l

If k is a positive integer,(1) i m p l i e s that


Fn + k = F n-1 _ + k + Fn-2~
Hence for the set (A)
F-n + p,
^1 = F n-1, + rp,l + F n-^.
P = F n-1, + q^ + F n-29
P = F

+q

Comparing the last equation with (2) and (3) we see that the r e p r e s e n t a t i o n of an i n t e g e r
of a n i n t e g e x

Q m a y be converted into a r e p r e s e n t a t i o n

P of the set (A) by r e p l a c i n g

in the f o r m e r by F .

By this operation we m a y d e r i v e the r e p r e s e n t a t i o n s of F


the i n t e g e r s

P from the r e p r e s e n t a t i o n s of the i n t e g e r s

Q.

of

Deriva-

tion of the r e p r e s e n t a t i o n s of P in this m a n n e r leaves the n u m b e r of


Fibonacci n u m b e r s unchanged.

24

ON THE REPRESENTATION OF INTEGERS AS


We have now to c o n s i d e r the i n t e g e r s

February

P in the set (B).

We have
P = F +p 7 , p~ = F _, F ? + l , F ? +2, . . . , F _+(F . - F
-l)
n 2 ^2
n-2
n-2
n-2
n-2
n-1
n-27
= F +F ^+r, r = 0, 1, 2, . . . , F
.-1
n n-2
n-3
= ' F +R by (4)
The l a s t r e s u l t i m p l i e s that the r e p r e s e n t a t i o n s of the i n t e g e r s
P in the set (B) m a y be d e r i v e d from the r e p r e s e n t a t i o n s of the i n t e gers

R by adding F

to each of the l a t t e r .

This o p e r a t i o n i n c r e a s e s

by one the n u m b e r of the F . in the r e p r e s e n t a t i o n of P over that of


R from which it is d e r i v e d .
By these two o p e r a t i o n s the r e p r e s e n t a t i o n s of the F , i n t e g6 e r s in F <L P < F -, . can be d e r i v e d from the r e p r e s e n t a t i o n s of
n
n+1
^
the F ~, i n t e &g e r s in F , Q < F
and the F 0 i n t etog e r s in
n-2
n-1
n
n-3
F 7 < Q <F
..
n-2
n-1
The following equations follow from the above d i s c u s s i o n :
u

=u
n, m

,
+u ~
,
n - l , m n-2, m-1

(n > 2, m > 1)

= 0 for 2m > n.
n, m
These equations indicate that the u
m a y be r e l a t e d to the
J
^
n, m
binomial coefficients ( ) , which have the following p r o p e r t i e s :

( 5 ) = 0 for
Letting U
=(
,
to
n, m \ m -1
lations (5) with the U
n,
it possible to calculate

k > r.

), t h e s e r e l a t i o n s for t h e ( , ) become the r e v


/'
k '
substituted for the u
. Since (5) m a k e s
m
n, m
any u
with n > 2 and m > 1, t h e s e r e r
* n, m
,
lations c h a r a c t e r i z e the u
and s o u
= U
=(
, j for
n, m
n, m
n, m \ m - 1 /
n > 2 and m > 10

1965

SUMS OF FIBONACCI NUMBERS

25

The r e a d e r is r e f e r r e d to the p a p e r "A Combinational P r o b l e m "


by Lafer and Long in the November 1962 i s s u e of the A m e r i c a n Mathe m a t i c a l Monthly for an e x p o s i t o r y account of the inductive and deductive a s p e c t s of a s i m i l a r p r o b l e m .

[3]

The proof of this is left to the r e a d e r .


We t u r n now to a d i s c u s s i o n of m a x i m a l r e p r e s e n t a t i o n of i n t e g e r s a s s u m s of Fibonacci n u m b e r s .

In this d i s c u s s i o n we shall use

a different technique,. one that could have been used equally well in the
d i s c u s s i o n of m i n i m a l r e p r e s e n t a t i o n s .
As an example we c o n s i d e r the i n t e g e r s
N < F0-l.
o

N such that F -1 <.

These a r e , 13, 14, 15, 16, 17, 18, 19 and 20.

for using the range

F -1 <, N < F

The r e a s o n

instead of F ? < N < F g

will

become evident l a t e r .
B e a r i n g in mind the definition of m a x i m a l r e p r e s e n t a t i o n we
d e r i v e the following r e p r e s e n t a t i o n s
12

F 6 + F 4 + F 2 ; 13 = F 6 + F 4 + F 3 ; 14 = F 6 + F 4 + F 3 + F2;

15 = F 6 + F 5 + F 3 ; 16 = F ^ F ^ + F ^
18 = F 6 +F 5 +F 4 +F 3 ; 19

17=

F ^ + F ^ ;

6+F5+F4+F3+F2

These eight r e p r e s e n t a t i o n s m a y be w r i t t e n compactly in the


following f o r m .

12

13

14

15

16

17

18

19

26

ON THE REPRESENTATION OF INTEGERS AS

February

In this display we have used the b i n a r y digits in conjunction with


Fibonacci n u m b e r s denoting place v a l u e s .

It should be noticed that

with this s c h e m e two z e r o s cannot be in adjacent p l a c e s in m a x i m a l


representation.
(.,.011..o)

For

since

example

(. . . 100. 0 . )

F. = F. , + F.
i-l

o0

must

be r e p l a c e d

by

Also the Fibonacci n u m b e r s d e -

i-2

noting the positional values a r e a r r a n g e d in ascending o r d e r from the


right to left beginning with

F?.

We now c o n s i d e r the g e n e r a l c a s e .

Let N be an i n t e g e r defined

by
F -1 <. N < F , , - 1
n
n+1
Let V
n, m
require m

denote the n u m b e r of i n t e g e r s N in this i n t e r v a l which

Fibonacci n u m b e r s to r e p r e s e n t them in m a x i m a l r e p r e -

sentation.
Thus for the i l l u s t r a t i v e example given above
v
= 3* V
=4-V
=1
v
7, 3
' 7,4
' V 7, 5
Also
V
+F
+V
= F -F = F, = 8
V
7, 3 7,4 V7,5
*8
7
6
The l a r g&e s t i n t e &g e r in the i n t e r v a l F n -1 < N<F n+1
^,-1
and since (2)

is

F n+1
^,-2

n-1
2
^

F. = F , ? - 2
1
n+2

i=2

it follows that
F

-2 = (111. . .11)

(n-2 digits)

in which no z e r o s a p p e a r and in which the left hand positional value is


F

, . This explains the r e a s o n for taking the upper bound of N to be

F n+1
,,-1

instead.of

F n+1
.,.

The s m a l l e s t i n t e g e r in the range in question is

F -1 and since

1965

SUMS OF FIBONACCI NUMBERS

27

n-2
V

F. = F -2 < F -1
i n
n

i=2
it follows that there must be a "one" in the first (left hand position) defined by F
J

,.
n-1

Further since (2)


v
'
F 0 +F / 1 +F,+. . . +F = F ,, (n even)
2 4
6
n
n+1
'
F Q +F-+F+. . . +F = F M (n odd)
3
5 7
n
n+1

it follows that the smallest integer in the range in question is indicated


by
(1010. . . 10) or
according as

(1010...101)

n is odd or even.

From these observations we conclude that


/ m > n- 2 or
V

'

= 0 if
~
\m

< p ^ ]

n < m+2

or

n>2(m+l

n-2
I
n-1

-[]
i

V . = F X1-F = F .
n, I
n+1
n
n-1

Table II gives values of V

for n = 2, 3, . , . I 2 ; m ~ l s

2, . , . , i o .

We now establish the recursive relation


(6)
K
'

V ' = V .
,+V 9
,
n, m
n-l,m-l
n-2,-m-l
Consider integers P, Q and R defined by
F -1 < P < F , . -1
n
n+1
F

. - 1 <- Q < F - 1
n-1
n

n-20

-l < R < F

.-1
n-1

The Fibonacci positional representation of the integers


the type

Q are of

28

ON THE REPRESENTATION OF INTEGERS AS


F

n-2
Q = ( 1

n-3
a

February

n-4
" 2
b .. . c )

Adding F

, to each i n t e g e r Q will produce F _ i n t e g e r s


r
n-1

n-2

all of which will be within the i n t e r v a l


F ,+F , - l ^ . Q + F , < F
_+F -1 .
n-.l
n-1
n-1
n-1
n
&

This is equivalent to
F

,, - F +F . -1 < Q + F . < F ,. -1
n+1 n n-1
n-1
n+1

F n+1
,,-1
X1-F
n-1, - F n - 27 + F n-1, - l < Q + F n-1! < F n+1
F

_F
-1 < Q + F
< F
-1
n+1 n-2
- W
n-1
n+1
F +F . - 1 C Q + F . < F , . - 1
n n-3
n-1
n+1

These
F

Q + F , a r e all in the i n t e r v a l
0 integers
n-2

n-1
, - 1 . Their positional r e p r e s e n t a t i o n t a k e s the form
n-1

n-2

n-3

Q +F

. = ( 1
1
a
n-1
Hence the r e p r e s e n t a t i o n s oi F
d e r i v e d from the i n t e g e r s
as

F n-1, .
The

n-4

F -1 < P <
n

b ' . . .. c )
?

of the i n t e g e r s

P m a y be

Q by c r e a t i n g an additional position defined

integers

R have positional r e p r e s e n t a t i o n s of the

form
F
R = (
Addingto F

. F
F . . . . F7
n - 3 n - 4A n-5
2
1

, to each of t h e s e
n-1
all in the i n t e r v a l

d
F

e
n - 30

...

f )

i n t e g e r s will r e s u l t in i n t e g e r s

,+F -l < R + F , < F


,+F
.-1
n-1
n-2
n-1
n-1
n-1
F -1 < R + F . < F _L1 - F
-l
n
n-1
n+1
n-10

1965

SUMS OF FIBONACCI NUMBERS

That i s , t h e s e
F

. -1.

29

i n t e g e r s a r e all within the i n t e r v a l F -1 < P <


n-o
n
Each of them will have r e p r e s e n t a t i o n s of the form
n-1
R + F

n-2

n-3

n-4

n-5 e o

.
n-1

Hence the r e p r e s e n t a t i o n s of F

P m a v be
0 of the i n t e g e r s
&
J
n-3
obtained from the r e p r e s e n t a t i o n s of R by adding on the left two p o s i tional values n a m e l y F , and F ~.
J
n-1
n-2
Since the f i r s t operation r e s u l t s in a r e p r e s e n t a t i o n which h a s a
r

" o n e " in the second (from the left) place while the second o p e r a t i o n
gives a r e p r e s e n t a t i o n with a z e r o in that place the two r e p r e s e n t a t i o n s
a r e disjoint.

Thus t h e r e is no overlapping and all i n t e g e r s

P are ac-

counted for by t h e s e two o p e r a t i o n s .


This c o m p l e t e s the proof of (6).
It is r e a d i l y verified that

(7)

=(

-)

.
n, m
Vn-m-2 /
s a t i s f i e s the r e c u r s i v e r e l a t i o n (6).
F r o m (7) we find that
2(m+l)

v ^ . ()(?)() + ...()

i=m+2
Also from the p a r a g r a p h following (5) and (7) we see that
V

n. m

= U

n, n - m - 1

REFERENCES
P . Lafer, "Exploring the F i b o n a c c i R e p r e s e n t a t i o n of I n t e g e r s , "
The Fibonacci Quarterly^ A p r i l 1964, p . 114.
Cf. N. No Vorob'ev,

Fibonacci N u m b e r s , New York, 1961, pp.

6, 70
P . Lafer and C. T. Long, "A C o m b i n a t o r i a l P r o b l e m , "
A m e r i c a n M a t h e m a t i c a l Monthly, Nov e 1962, pp. 8 7 6 - 8 8 3 .

The

30

ON THE REPRESENTATION OF INTEGERS AS


Table I
n = 1, 2, . . . , 8

Values of U

1 "
[

Km]

F ,I I N ^ ' F ,L j|

1N<F

F6"lN<F7
|

i N C F

fFgVN < F

'
/:

m == 1, 2, . . . , 4

1 1 2 j -3 | 4

_0 u! 0 4X
| 0 | 0 Jj

PFTYN < F ~r i
1 3
rv32 < N <iYp
\ F, SN < Fc

February

11

oI

0 |

11 1
j

2 j 0

l"j 3] 1

r Tt

T r r

7
8

| 5 | 6 !
i 1
I

'

Table II
Values of V

n = 2, 3, 4, . . . , 12;
3

F2-l < N <F3-1


FQ-1 I N < F , - l
D

F.-l I N
4

4:

1 I 0 j 0 I 0

<F_-1

0 J

8
0

1 j 1 S CM 0

0 I

F 6 - l 1 N < F -1

6 I 0

0 I 3

F 9 - 1 < N <F10-1

^O" 1 ^ N ^ l l - 1 )

10

0 I 0 J1

T{ 0 f 0

F _ - l < N < F 1 0 - l | 12 | 0
i

o I oI oj

i_
J 10 j

0 * 0 ) 0

10

o so I o

F 1 -1 1 N < F 1 7 - l [ i l l
13

4
I

0 | 0 j 4
I

0 { 0 )

0 | 0 j 1 i 6

<F9-1

o i o i o

1 I 3 | 1

F ? - l <_ N < F g - l

10

12

0 \ 0

F 5, - 1 N < Fo, - l

F8-l N

m = 1, 2, 3, . . . , 10

-.-1~

10 j 15

o
l

21 j 8 t 1
5 | 20
\
1
15 j

N. B.
The e n t r i e s in the v e r t i c a l columns a r e r o w s of PASCAL'S
a r i t h m e t i c t r i a n g l e so that the table m a y be e a s i l y extended.
XXXXXXXXXXXXXXX

ON A GENERAL FIBONACCI IDENTITY


JOHN H. HALTON
University of Colorado, Boulder, Colorado

1.

The Fibonacci sequence is defined by the recurrence relation

<L>

n+2

= F

+ F

together with the particular values


(2)

FQ = 0, F 1 = 1

It is easily verified that the unique solution'1' of (1) and (2) is given by

F n = (a11 - p n )/(a - p) ,

(3)
where

a and (3 are the roots of the equation


x2 = x + 1

(4)
namely

1 ( 1 + ^5), p = 1 ( 1 - / 5 )

(5)

The sequence is thus defined for all integers


or zero.

n, positive or negative

From (1) and (2), we infer that (3) takes integer values for

all ns and we observe, by (3) and (5), that


(6)
x
'

= (-l)n+1F

F
-n

.
n

This sequence and its generalizations have been the subject of a


vast literature, and a very large number of identities of different kinds,
involving the Fibonacci numbers, can be demonstrated.

It is the pur-

pose of this paper to show how a considerable body of these may be obtained as particular cases of a single identity.

Direct substitution shows that (3) is a solution of (1) and (2),


1

If

F'

were another solution, f = F - F


would satisfy a relation (1), with
J
v
n
n
n
"
fA - f1 = 0. Induction on n now shows that f = 0 for all n, so that
0
1
n
(3) is the unique solution, as stated.
31

32
2

ON A G E N E R A L F I B O N A C C I I D E N T I T Y

February

We b e g i n by d e f i n i n g t h e f u n c t i o n

(7)
x
'

Sn(n) = F
O
n

Then, i m m e d i a t e l y ,

+ F

,. - F , 9
n+1
n+2

by (1), w e s e e t h a t , f o r a l l i n t e g e r s

(8)

S Q (n) = 0

n,

Now c o n s i d e r t h e f u n c t i o n
(9)

S, ( m , n) = F F + F
, . F ,. - F
, ,.
1
m n
m + 1 n+1
m+n+1

T h e n , by (1), f o r a n y
(10)
Also,

and

n,

S ^ m + 1, n) = S ^ m ,

n) + S ^ m - 1, n)

by (1), (2), (7), a n d (8), w e h a v e t h a t


S L ( 0 , n) = F n + 1 - F n + 1 = 0

(11)
and

S^l,

n) = S Q (n) = 0;

w h e n c e (10) y i e l d s , by u p w a r d a n d d o w n w a r d i n d u c t i o n o n m , t h a t ,
all integers

and

for

n,

(12)

Sjfm,

n) = 0

Next c o n s i d e r the function


(13)
v
'

S , (Xr ,
2

m , n) = F F - ( - l ) f v( F
, F , - F F
, , )
'
m n
* '
m + r n+ r
r m+n+r

A g a i n a p p l y i n g (1), w e s e e t h a t
(14)

S 2 ( r + 1, m , n) = S ( f - 1, m , n+2) - S 2 ( f, m , n + 1)

Now, f o r a n y

or

n, by (2), (9), a n d (12),

So(0, m,
2'

n ) = F F - F F = 0
m n
m n

(15)
and

S?(l,

m , n) = S, ( m , n) = 0

We m a y a l s o n o t e t h a t , f o r a n y f i x e d
(1).

n, (10) i s a r e l a t i o n of t h e f o r m

T h u s , a s i n t h e p r e v i o u s f o o t n o t e , w e g e t (12), f o r a l l

and

n.

1965

ON A GENERAL FIBONACCI IDENTITY

Thus, by upward and downward induction on


for all i n t e g e r s r , m, and n,
(16)

33

r in (14), ' we find that,

S 2 (r, m, n) = 0 .

Finally c o n s i d e r the function (with k > 0)


k
k

(17) S~(k, r , m, n) = F F - { - l )
3
m n

kf

1 ( k ) ( - l ) h F ^ F k T h F ,. ,,
x ,x
n
'
r f+m n+k r+hm
h=0

It is well known that

(18)

+ (

<h > = 0

h-l}

and
(19)

() = 0 when h < 0 or 0 < k < h

Thus we can show, by (13), (16), (18), and (19), that


(20)

S 3 (k + 1, r , m, n) = ' F m S 3 ( k , r, m, n)

Also, by (13), (16), (17), and (19),


S(0, r ,
3

, n) = F

- F

=0

(21)
and

J S~(l, r , m , n) =

S^(T3

m, n) = 0

Thus, by upward induction on k in (20), we get that, for all i n t e g e r s


r, m, and n, and all i n t e g e r s
(22)

k 2 0,

S 3 (k, rs m, n) = 0

We o b s e r v e that, while the inductive a r g u m e n t leading to (12) a s s u m e s


an a r b i t r a r i l y c h o s e n a n d fixed n; the c o r r e s p o n d i n g a r g u m e n t y i e l d ing (16) a s s u m e s , at each step, that (16) holds for a consecutive p a i r
of values of r , an a r b i t r a r y fixed value of m, and all values of n.

ON A G E N E R A L F I B O N A C C I I D E N T I T Y

34

February

This is the g e n e r a l identity p r o m i s e d above:

FkF = ( - D k r s

(23)

m
3.

'

(5(-i) h F>^: h F

h=0

h/v

'

r+m

+v +.

n+kr+hm ?

We m a y n o w l o o k a t s o m e of t h e i d e n t i t i e s w h i c h a r e o b t a i n e d a s

p a r t i c u l a r c a s e s of ( 2 3 ) .

On t h e left of e a c h i d e n t i t y b e l o w , t h e v e c t o r

(k, r , m , n) i s s h o w n .
move negative

In s o m e c a s e s , t h e i d e n t i t y (6) i s u s e d t o r e -

subscripts.
k
= 2 ( k )x( _
** V
h=0

(24)
(k, r , - m , - n ) : F k F
x
' x ' '
'
'
m n

1}(m-1

M k - h ^ k - h
f r~m

'

n-kr+hm

/ o c \ /i
i
\ i-k
v /^\/ i x ( m - l ) ( k - h ) _ h _ k - h
F F
F
(25) (k, f, - m , - k r - n ) : F F ,, = Z (, ) ( - l )
* ,.
n+hm
x
'
m n+kr
r r.-m :
h=0 x h

(26) ( k , r , m , - k r ) : F ^ F k r =

(JX-D^^^F^

h=l

(27) (k, r,m,m): F k + 1 = ( - l ) k f 2 ( 5 ( - l ) h F ? F k ; h F,


v

.'

'

'

'

ri

'

r+m

,,,,_.

kr+(h+l)r

h=0
(28)
(k, r, m , n r ) : F k F
= ( - l ) k f 2 v( k /)v( - 1 ) h F h F k ~ h F , ,. . ,,
v
' v ' '
'
'
m nr
'
h
'
r r + m (n+k) r + h m
h=0

(29)
(k, r , m r , n ) : F k F = ' ( - l ) k f 2 ( k ) ( - l ) h F h F / k T ^ F ..*.,'
,
N
' v ' '
' '
mr n
'
h
'
r ( m + l ) r n+(k+hm)r
h=0

k
,x/
(30)
) h F hr F krT+ hm F , k r +,,h m = 0
\
/ (k,
\ >r ,9m , 0 )j: 2 (xfa)(-J.
h=0

(31) (k. r.m,l); F ^ = (-l> k ' 1 <)<-1 ^


h=0

r+hm1

1965

35

ON A GENERAL FIBONACCI IDENTITY


k

(32)

(k,r,mr,0):

2 ^ - ^ V ^ / J ^ , F/lr+, *
n
r ( m + l ) r (k+hm)r

= 0

k
1

(33)

( k . r . l . n ) : F R = (-1) " 2 ^ ( " D ^ F ^ F ^ ^


h=0

(34)

(k.,.l.-kr-n):Fn+kf=

S & F J F ^ F ^

k
(35)

<k.r.l,-kr):Fkr =

S ^ F ^ F ^ F ^
h=l
k

(36)

(V,*l.nr): F ^ = ( - i f

2
( f r - U ^ F ^ F ^ , ^
h=0

(37)

( k . r . 2 . n ) : F n = <1 ) k ( - 1 ) k ' 2

> ( - n ^ ^ n + k ^ h

k
(38)

(k, r , 2 , - k r ) : F ^ = ( l )

2 ()(- 1 ) h " l F > ^ F

2 h

h=l

(39)

(k.l.mfn,:F^Fn=(-Dk2

fy-^^nk+hm

h=0
(40)

(k)l,m)?k): F ^ F ^ (Dk 2 # < - 1 ^


h=l

(41)

(k,2,m,n): F ^

= 2
h=0

'

ft^F^F^Zk+hm

36

ON A GENERAL FIBONACCI IDENTITY


k

(42)

(k.r^l.0):

2(^(-l>h^;jFkrh=0
h=0
k

(43)

(k,r, 2,0): 2

fy-^>^krZh

= 0

h=0
k

(44)

(k.il.m.O):

S(J)(-DhFmkXk=0
k

(45)

( k , l , m , . l ) : F ^ = (-l) 1 (^)(-l)hF^Vhmk11
k

(46)

(k.l.l.-n):FaS=

X <)F n _ k _ h
h=0
k

(47)

(k,l,l,-nk):Fnk=

X <*>F(n_1)k_h
h=0
k

(48)

(k, 1 , 1 , - k ) : F k =

(49)

(k, 2, - 1 , - n ) : F n ."

2 (^)(-l)h""1Fh

h)Fn-2k+h

h=0
k
(50)

(k, - 1 , 2 , - n ) : F n =

^-^n+k-Zh

h=:0

(51)

(k, 2, - 1 , -2k): F
h=0

February

1965

ON A GENERAL FIBONACCI IDENTITY

37

k
(52)

(k, - l , 2 , k ) : F k =

X (hH"1)k'hF2h
h=0

k
(53)

(k, 1,1,0):

X (hX-^^k+h2
h=0
k

(54)

(k,2, -1,0):

2(h)(-1)hF2k-h=0
h=0

(55)

(k, - 1 , 2 , 0 ) :

(hJ<-1)hFk-2h=
h=0

(56)
\ /

(\ l ,r ,>m , 9n):/ F F n = ( - l ) f ( F m +, r F n+, r - F r F m+n+r'


, . )
m

(57)
v
'

(l,r,m,-n):F F - F X F
=(-l)n~fFF
,
* '
m n
m + r n-r
r m-n+r

(58)

( l , r , m , -m): F 2

(59)
x
'

v 5

(60)
v
'
(61)
v
'

- F

^ F

=(-l)m"rF2

( l , r , m , n-r): F F , = F ^ x F - (-l)fF F
' '
'
r m+n
r+m n
m n- r

(1, r - k , m+k, -m+k): F , J


. -F
, F _
= ( - l ) m ~ f F F,
'
'
'
'
m+k m - k
m + r m-r
r k+r
, ., = Fm F n + F m .,.F
.,
x( l , l , m , n ) : F
'
'
'
m+nl
l ni

x(62)

x(1,

(63)
v '

v(1,

(64)

(x l , l , m , m ) : F 9 ,, = F 2 + F 2 .
'
2ml
m
ml

(65)

(1,
l,m,m-l): F9
=F-(F
,, + F
.)
x
2m
m m+1
m-1

(66)

(l,2,m-l,m-l): F9
= F 2 ,, - F 2 ,
2m
m+1
m-1

'

^ = F m+1
, , Fn + F mF n-1,
' l ', m ,' n - 1 )' : F m+n
2, m - 1 , n - 1 ) : F ^ = F , . F ,. - F
,F ,
' '
'
'
m+n
m+1 n+1
m - 1 n-1

38

ON A GENERAL FIBONACCI IDENTITY

(67)
x
'

(1, l , m , -m): F 2
\ > > >
m

(68)
x
%

(1,2, m, -m): F 2 - F
, F
= x( - l ) m
m
m+2? m-27
'

(69)

(1, l , m + l , -m+1): F

-F

February

,. F
=(-l)m"1
m+1 m-11

,. F
_-F
,7F
- .= 2 ( - l ) m
m+1 m-1
m+2 m-2
'

This rather long list includes most of the identities, derivable


as special cases of (23), which I have found in the literature, and a
number of others (including (23) itself, (24)-(32), (37)-(45), and (60)),
which I believe to be new and useful. ^
4.

We may now ask what else can be done with the family of identi-

ties (23)-(69). Some of the further developments will be demonstrated


belowo
Putting

n =m

in (59) and dividing by F

, we obtain,

by (65)

and (6), that


m

,. + F
JF = F ,
+ (x - l ) i i avF
m+1
m-1 r
r +m
'
r- m

(70)
x
'

v(F

Thus
( 7 1 )|~F x 1 + F
. - 1 - { - l ) m ] F r = (F j_ - F ) - ( - l ) m ( F r - F
) .
v
v
X
'L m+1
m-1
J T
r +m
T'
' s r
r-m'
The usefulness
sum from

of this identity is seen when we put

r = 1 to r = t.
t

,_.
(72)

_, _
2 Fr
r=l

F
m + n

r = rm + n and

The right-hand side telescopes to yield

- F
(-l)m(F
- F )
(t+l)m+n
m+n
tm+n
ny
m
F
+F
- 1 - (-l)
K
}
m+1
^m-1

(This result is known [1] , but I believe that the line of proof is new. )
Certain particular cases have been knownfor a long time; for instance,

EDITORIAL NOTE:

A different form of the identity (23) appears in

an unpublished Master's Thesis entitled "Moduls

m properties of the

Fibonacci numbers, "written by John Vinson at Oregon State University


in 1961. (Other parts of that thesis appear as a paper by John Vinson
in the Fibonacci Quarterly,

1(1963) No. 2, pp. 37-45.)

1965

(73)
N

ON A GENERAL FIBONACCI IDENTITY

I F

39

, = F. , 1 , 9 - F. ... + F. , - F = F + , ^ - F ^ ,
r+s
t+1+2
1+s
t+s
s
t+s+2
s+2

r=l '
t
(74)

X F 2 ( r + S ) = F 2(t+l+s)~ F 2(l+s)~ F 2 ( t + s ) + F 2 s
r=l

(75)

2 F2(r+s)-l "
r=l

2(t+s) "

2s

= F

2(t+s)+l "* F 2s+1'

and
t
2

(76)

3(r+s)

4 (F 3(t+l+s) ~ F 3(l+s)

r=l
+ F
- F ^ = ( F
r
*3(t+s)
3 s ' 2ir3(t+s)+2
If w e s u m (64) f r o m

- F
)
r
3s+2;

m = s + 1 t o m = s + t , p u t r = m - s, u s e

(75), a n d s l i g h t l y r e a r r a n g e t h e r e s u l t , w e o b t a i n t h a t
t
< 77 >

r+B=-i<F2(t+fl)
r=l

+ F

t+B-F2B-FB)

Now rewrite (65), using (1), in the form


(78)
x
'

F0
F 2 = 2F F J_.
2m
m
m mi

and sum (78) as before, using (74) and (77); then we get

(79)

I F r + s F r + s + l ~ 4^F2(t+s)+3
r=l

+ F

t+s " F 2s+3 " F s ^

If we sum (7 3) with t = w - s, from


that

s = v to

'

s = w - 1, we get

40

ON A GENERAL FIBONACCI IDENTITY


w-1

w
F

u-1

2 F

u
s=v

(u-v)F

u=s+l
- vF

u=v+l s=v
w-1

,, =
w+2

2 (F

February

='

u
u=v+l

uF

+ vF
u

x, v+2

u=v+l

, - F , . ) = (w - v)F x 9 - F , Q + F ^ ,
w+29
s+2
w+2
w+3
v+3

s=v

which yields
w

(80)

uF

= wF
u

, 0 - F , Q - vF ,, + F ^
w+2
w+3
v+2
v+3

u=v+l
The s a m e p r o c e s s of s u m m a t i o n applied to (80) yields
w

(81)
x
'

u 2 F = w 2 F , 7 - ( 2 w - l ) F . 4 ~ + 2 F , A + ( 2 v - l ) F ,- - 2 F ,,
u
w+2 s
' w+3
w+4 v
' v+3
v+4

2
u=v+l

and we can evidently i t e r a t e the p r o c e d u r e to obtain the sum of u


for any positive i n t e g e r

m.

Again, r e p l a c e m by

r + m in (63) and apply (61) to the r e s u l t .

This gives
F

= (F
r+m+n

F
+ F F ) F
(FF
+F
F
)F
r+1 m+1
r m n+1
r m
r-1 m-1
n-1

or, by (1),
(82)

r + m + n

=F

r + 1

m + 1

n + 1 +

FrF

Fn-F_1F

_1Fn_

In p a r t i c u l a r ,
3
(83)
v

= F
3m

3
+F

m+1

3
- F

m-1

and
(84) F
= F F
F
+F
F F
- F
F
F
*
3m
m m+1 m+2
m - 1 m m+1
m-2 m-1 m
We m a y note, at this point, that (83) can be put in yet a n o t h e r f o r m ,
with the help of (67):

1965

ON A GENERAL FIBONACCI IDENTITY

41

F~
= F 3 + (F ,. - F
. ) ( F 2 , T + F ,. F
. + F2 . ) =
3m
m
m+1
m - l / x m+1
m+1 m - 1
m-1'
= F

+F

[~(F
- F
) +3F
F
1
m L m+1
m-1
m+1 m ~ U
F 3 +F ( F 2 + 3 [ F 2 + ( - l ) m ] l
m
m ( m
Lm
J J

in

=
or
(85)

F~ = 5 F 3 + 3 ( - l ) m F
3m
m
m
By s u m m i n g (83) and (84) from m = s + 1 to m = s + t, and using (76),
we obtain r e s p e c t i v e l y that
t
(86)
F3
= i ( F _ . , x , - 2F3
. - F . . + 2 F 3 .)
^ r+s
2' 3(t+s)-l
t+s-1
3s-l
s-1'
r=l
and
t
x
(87)
'

2 F r+
, F r+s
, F r+s+1
, ,,=
x s-1
r=l

If we m u l t i p l y (67) by F

2 F 3r+s
, + ( - l ) t + S F .t+s-1
,
, - (v~ 1 ) S F s-1, .
r=l

and sum from

m=s+l

to m = s + t, we

get that
t

r+S
1
2^ F r+s
,, - S
^ F r+s
,
x __
^ F r,+ s - 1i F r+s
+, 1 = 2 (x- l )' " " F r+s
r=l
r=l
r=l

A c o m p a r i s o n of this l a s t r e s u l t with (87) yields


(88)

t
2(-l)r+SFr+sM-Dt+SFt+s.1-(-l)SFs.1

r=l
This last r e s u l t m a y b e verified by combining (73) and (74), or by s u m ming the identity (derived from
(89)

(l))9

(-Dr+SFr+s = (-l)r+S"2Fr+s.2 -

( - D ^ - ' F ^ . J

As a final i l l u s t r a t i o n of the l a r g e family of identities springing


from (23), we c o n s i d e r the g e n e r a l i z a t i o n s of (66) and (83), analogous

42

ON A GENERAL FIBONACCI IDENTITY

to that of (1) in (70).

February

F i r s t we obtain a few results analogous to (85).

Clearly
2
(F

+F

m+1
m-r
thus, by (1) and (67),

= (F
l

- F
m+1

?
) + 4F
F
m-l}
* m + l m-1

,. + F
. ) 2 = 5F2 + 4 ( - l ) m
m+1
m-1
m

(90)
w
'

v(F

'

and therefore, by (65),


F2
- 5F4 + 4(-l)mF2
Zm
m
m

(91)

Also, by (I),
(92)
'

,. + F
= F
- F
= I(F ,% + F
)
n+1
n-1
n+2
n-2
2 n+3
n-3

whence, by (85) and (67),

<93>

Putting
(67),
(94,

3m+1

2r

+F

for

3m-1 = <Fm+l

f and 2m for

+F

r-1> [ 5 F L

m in (70), we get, by (64), (66), and

that
[ 5 F ^ + 2 ( - l , m ] . F 2 r = FZT+m+l

- F

F2_m+1 - F

. ^

Alternatively, on squaring (70), we obtain, by (58) and (90), that


r 5 F 2 + 4x( - l ) m > 2 = F*
+F2
+ 2 ( - l ) m r F 2 - v( - l ) r + m F 2 J1 ,
L m
' J 7
r+m
r-m
L r
'
m
whence
(95)
x
'

T5F2
L m

+2(-l)m~|F2 + 2(-l)rF2
X /
x /
J r
m

= F2
+ F2
r +m
r-m

We see that (94) yields (66) on putting one for


nally, put
we get

3r

for

r and

3m for

m and m for

r .

Fi-

m in (70). Then, by (85) and (93),

1965

ON A GENERAL FIBONACCI IDENTITY

43

2
3
(F m+1
,. + F m - 1,)' [L5 F m
+ v( - l ') m ]J F Q3r = 5 F f+m
+ 3v( - l' ) f + m F r+m
, +

+ 5(-l)mF3
+3(-l)rF
s
r-m
'
r-m
3
m
r+
^5Fr +m+5(-l) Fj_m + 3(-l) (Fm+1+Fm_1)Fr
=

5Fj+m+5(-l)mFr3_m

(-l)m(Fm+1+Fm_1)(F3r-5F3)

Thus, by (65),
3
F F . = F 3 ^
- (-l)m(F
., + F
+ (-l)mF3
1)F
m 2 m 3r
f+m
m+1
m-1 r x '
r-m
This identity is new, but we can find in the l i t e r a t u r e [2] the p a r t i c u l a r

(96)

c a s e s when m = 1 and m = 2, n a m e l y (83) (with f for


3F

(97)

3r

= F

2 "

3 F

+ F

l 2

m) and

REFERENCES
1.

K. Siler, " F i b o n a c c i S u m m a t i o n s , " Fibonacci Q u a r t e r l y , 1(1963)

2.

No. 3, pp. 6 7 - 6 9 .
Fibonacci Q u a r t e r l y , 1(1963) No. 2, p. 60.
XXXXXXXXXXXXXXX

REQUEST
The Fibonacci Bibliographical R e s e a r c h Center d e s i r e s that any
r e a d e r finding a F i b o n a c c i r e f e r e n c e , send a c a r d giving the r e f e r e n c e
and a brief d e s c r i p t i o n of the c o n t e n t s .

P l e a s e forward all such in-

f o r m a t i o n to:
Fibonacci Bibliographical R e s e a r c h Center,
Mathematics Department,
San Jose State College
San J o s e , California

ADVANCED PROBLEMS AND SOLUTIONS


Edited by VERNER E. HOGGATT, JR.
San Jose State College, San Jose, California

Send all c o m m u n i c a t i o n s concerning Advanced

P r o b l e m s and

Solutions to V e r n e r E. Hoggatt, J r . , M a t h e m a t i c s D e p a r t m e n t ,
J o s e State College, San J o s e , California.

San

This d e p a r t m e n t e s p e c i a l l y

w e l c o m e s p r o b l e m s believed to be new or extending old r e s u l t s .

Pro-

p o s e r s should submit solutions or other information that will a s s i s t


the e d i t o r .

To facilitate t h e i r c o n s i d e r a t i o n , solutions should be s u b -

m i t t e d on s e p a r a t e signed s h e e t s within two months after publication


of the p r o b l e m s .
H-52

Proposed by Brother U. Alfred, St. Mary's College,

California

P r o v e that the value of the d e t e r m i n a n t :


n
2
un+2
2
un+4
,n+l
is 18 (-1)
H-53

un+2
2
un+4

"n+4
2
U
n+6

2
un +6

n+8

Proposed by V.E. Hoggatt, Jr., San Jose State College, San Jose, California
and S.L. Basin, Sylvania Electronics Systems, Mt. View, California

L ,. + L
for
The Lucas sequence, L, = 1, L ? = 3; "n+2
n+1
n
n -^ 1, is incomplete (see V. E. Hoggatt, J r . and C. King, P r o b l e m
E - 1 4 2 4 A m e r i c a n M a t h e m a t i c a l Monthly Vol. 67, No. 6, J u n e - July 1 960
p . 593) since e v e r y i n t e g e r
bers.

n, is not the sum of d i s t i n c t Lucas n u m -

OBSERVE THAT 2, 6, 9, 13, 17, . . . cannot be so r e p r e s e n t e d .

Let M(n) be the n u m b e r of positive i n t e g e r s l e s s than n which cannot


be so r e p r e s e n t e d .

Show

M(L ) = F
.
n
n- i
Find, if p o s s i b l e , a closed form solution for M(n).
H-54

Proposed by Douglas Lind, Falls Church, Va.

If F

is the nth Fibonacci n u m b e r , show that


*<F )

where

0 (mod 4),

<f)(n) is E u l e r ' s function.


44

n > 4

45

1965

ADVANCED PROBLEMS AND SOLUTIONS

H-55

Proposed by Raymond Whitney, Lock Haven State College, Lock Haven, Penn.

Let

F(n)

numbers,

and

L(n) denote the nth

Fibonacci and nth

Lucas

respectively.

Given U(n) = F(F(n)), V(n) = F(L(n)), W(n) = L(L(n))

and

X(n) = L(F ), find recurrence relations for the sequences U(n), V(n),
W(n) and X(n).
H ~ 5 6 Proposed by L. Carlitz, Duke University,

Durham, N.C.

Show

,
n=l
H-57

- ( V*W

TTTF
F
n n+28 * n+k n+k+1

~ "k+I

7f
i=i

Proposed by George Ledin, Jr., San Francisco,

If

is the

nth

k >,
>

"

_
F.
x

California

Fibonacci number,

define

G
n
and show
(i)
w
(ii)

lim x(G , . - G ) = 1
^
n+1
n
n~>
lim
(G n + 1 /G n )
= 1
n-> a.

Generalize.
H-58

Proposed by John L. Brown, Jr., Ordnance Research Laboratory, The Penn. State
University, State College, Penn.

Evaluate, as a function of n and k, the sum


F

2
. .. ,
x1+i2+...+ik+1=n
where

2 L +2 F 2i 9 +2
1
2

L , i ? , i~, . . . , i , + ,

9e # F

2L +2 F 2 i , , , 4-2
k
k+1

'

constitute an ordered set of indices which

take on the values of all permutations of all sets of k+1

nonnegative

integers whose sum is n


REPROPOSED CHALLENGE
H-22

Proposed by Verner E. Hoggatt, Jr., San Jose State College, San Jose,
CO

If P(x) =

n
i=l

CO

Tj-

(1 + x

) =

X
n=0

R(n) x n

California

46

ADVANCED P R O B L E M S AND SOLUTIONS

February

then show
R ( F ~ - 1) = n
2n
'
R(N) > n if N > F 0 - 1
2n

(i)
(ii)

INVERSION O F FIBONACCI POLYNOMIALS


P-3

Proposed by Paul F. Byrd, San Jose State College, San Jose,

California

i n " E x p a n s i o n of A n a l y t i c F u n c t i o n s i n P o l y n o m i a l s A s s o c i a t e d

with

F i b o n a c c i N u m b e r s , " V o l . I, N o . 1, F e b . 1 9 6 3 , p p . 1 6 - 2 9 .
Verify the r e c i p r o c a l

relationship

[n/2]
X

= <L n 2
2

( - l ) r (n) Hl2+i
x

r=0

n-r+1

n+l-2rv

(x)

( n

'

Q )

'

[k/2]

where
X k + 1 (x) =

(kmm) ( 2 x ) k ' 2 m

( k > 0)

m=0
Solution by Gary McDonald, St. Mary's College, Winona, Minnesota

Verification by induction:

(Equation n u m b e r s

refer

t o P F . B y r d ! s

article,
For

n = 0, w e h a v e

\ - Y (x) w h i c h a g r e e s w i t h (2 2 ) . A s s u m i n g (1)

t r u e f o r n = k, w e c a n w r i t e

[k/2]
x

k+1

1
TFFT

^
^
r =0

'

i x
l )

r .k x k - 2 r + l
^-ETFFT

,-, x v
( 2 x ) y

, s
k+l-2r(x)

R e c a l l i n g ( 2 . 1), w e h a v e

~[k/2]
k+1

1
,k+l

[k/2]
r k

X ( - l ) ( ) ^ l l l i i ^_
V

k - r +1

r =0

'k+2-2r

, x
(X)-

, -, , r . k k - 2 r + l
(x)
(-1) (J-TTT^XT-yL.
V k-r+1 ' k - 2 r

r=0

[k/2]
.k+1

yk+2(x)+2(-i)^)^iyk+2.2r

(x)

r=l

" [k/2]-l
r
(x)
- s
(-i)r(kr)^."' k - r +.t1 y,
k-2r
r =0

+ C

1965

ADVANCED PROBLEMS AND SOLUTIONS

where
C =(

'

[k/2] +l/ k \ k - 2 [ k / z ] + 1
\[k/2]/k- [k/2] + 1 r k- 2 [k/2] (x)

Letting j = r + 1 in he second X f r
[k/2]
k+1
1
rk+2(x)+ S ( - i
,k+l
r=l
[k/2]

or combining coefficients of

47

x k+1

) r ( ^ ) ^ ^ y k + 2 . 2 r (x) +

J . k . k+3-2j

y. (x)
[k/2]

(2)

"k+1 = Jn <! W >


l

+
r=l

s i-1''V

r - l ' k+2~r

k-2r+l
k-r +1

k+2-2r(x) c (

We can r e d u c e the quantity in b r a c k e t s a s follows:


/ k \ k " Z r + 1 +/ l k ^k+3-2r
V k-r+1
r-l'k+2-r
=

[^ M ar">^.""H(^

k
r
n 7 xnnj.9
x x k!(k+3-2r) "1 / 1 \
[(k-r+i)!r; ( k - 2 r + 1 ) ( k + 2 - r ) + ( F ^ + i r n ^ I T : J V k T 2 r 7 J

(k-2r + l)(k+2-r) + r ( k + 3 - 2 r ) ]
(k+1)!
k+1
J (k-r +1)1 r! (k+2-r)
(k+i):
|k + 3 k - 2 r k - 2 r + 2
(k- r+l)I ri (k+2-r)
L
(k+i)
k+2-2r / k + l \
k+2-r \ r ) '
T h e r e f o r e from (2),

(3)

k+1

1
,k+l

[k/2]
/ \ , v / -, x r .k+L k+2-2r v
x) + C
y k + 2 (x) + 2 (-1) ( r ) T T Z - F Yk+Z_2r
r=l

Note that:
a)

w ^ - ^ O S J ^ v2-oW

48

ADVANCED P R O B L E M S AND SOLUTIONS

b)

When

k is even

C = 0, a n d [ | ]

When

k i s odd, t h e n

C = (-1)

= (-1)
If w e l e t

(-1

r =

L 2 J

and

ese.

2 J 3+_k VAx)
'1
2

k - l v .k+1

3+k

y T (x)
I

i n t h e X of e q u a t i o n (3), w e h a v e

(k+l)i

2 y . . . .
k+3 V x ' ~ l

k + 1 . , ,k+l

= (-D
= C,
Therefore,

~- J =

= [^~]

February

1
'

2 J
(k+l)ki
2(k+l),k-l., ,k+I.,

[^]
k

odd.

we m a y c o m b i n e

Y,

_(x)

and

C into the

i n (3) a n d

write

[JH]
x

= -rrrr-

,k+l

(-1)

( _ ) , .9

r ' k+2-r

Y, . 9

(x)

'k+2-2rv

r=0
f r o m which we conclude

l>/2]

_L 2 i - i f fVl g ;n 2- r^+ 11 r.' n + l - 2 r (x),

n > 0

r:=0
D E F E R R E D ANSWER
H-34

Proposed by P.F. Byrd, San Jose State

D e r i v e the s e r i e s

V (x)

y,(x)

k+3

(^L)K^L):

4
k+7

College

expansions
OD

J 9? 1v x(a) = lha)
+
k% '
2k

2,
m=l

(-l)m+kI
.,(<*) I
, (a)
'
m+kv ' m - k

L,
2m

1965

ADVANCED PROBLEMS AND SOLUTIONS

49

( k = 0, 1, 2, 3,
) for the B e s s e l functions J
of all even o r d e r s ,
Zj
w h e r e L a r e Lucas n u m b e r s and I a r e modified B e s s e l functions.
n
n
The solution will a p p e a r in a fine p a p e r by the p r o p o s e r to a p p e a r
l a t e r in the Q u a r t e r l y .
FIBONACCI AND MAGIC SQUARES
H~ 3 5

Proposed by Walter W. Horner, Pittsburgh,

Pa.

Select any nine consecutive t e r m s of the Fibonacci sequence and


form the m a g i c s q u a r e
u

u?
u

'

show
U

8U1U6

+ U

3U5U7

+ U

4U9U2

u g u 3 u 4 +U l u 5 u 9 + u6u?u2
Generalize.
Solution by Maxey Brooke, Sweeny, Texas and F.D. Parker, SUNY, Buffalo,

If U

N.Y.

s a t i s f i e s the g e n e r a l second o r d e r difference equation, then


U,

un

u.

ur

U,

u_

ur

Ur

= 0

since

U , -, = &U ,, + SU with U, and U 0 a r b i t r a r y . The expann+2


n+1
^ n
1
Z
sion of this d e t e r m i n a n t yields p r o d u c t s whose s u b s c r i p t s add up to the

r e q u i s i t e 15 and yields the equality asked for in the p r o b l e m .


Also solved by the proposer.

G O L D E N S E C T I O N IN C E N T R O I D S
H-36

Proposed by J. D.E. Konhauser, State College,

Consider a r e c t a n g l e
move a r e c t a n g l e
of R.

R.

F r o m the upper right c o r n e r of R r e -

S ( s i m i l a r to R and with sides p a r a l l e l to the sides

D e t e r m i n e the l i n e a r r a t i o

remaining

Pa.

K= L / L c

if the centroid of the

L shaped region is w h e r e the lower left c o r n e r of the r e -

moved r e c t a n g l e w a s .

50

ADVANCED PROBLEMS AND SOLUTIONS

Solution by John Wessner, Melbourne High School, Melbourne,

February

Florida

G
O
GOr

CD =: L

OD = a L
OE = a L
D

T h e c e n t r o i d of A G P B
a L R \ L^ - L
y a

[.

is at

Similarly

* |L
the

Lq + L R I a n d h a s w e i g h t

r e m a i n d e r m u s t be a t

Z L S a IV L R!

2L

centroid

BEDC

is

at

weight
-j L ^ ,

T h e c e n t r o i d of t h i s

x-coordinate

HLS+LRI IVLRI

RILS-LRI

S -

gives
K

After

of

and h a s

-K-4I

or upon expanding

D i v i s i o n by

i.i .1 R

a L q j L~ - L p | .

and have

+ L

r e m o v i n g the obvious root

has as its positive root

y = (1 +

Editorial Comment:

- l

+1 w e h a v e

- K - 1 = 0

which

\]5)/2.

The above p r o p e r t y is s h a r e d by .many

g e o m e t r i c f i g u r e s including the ellipse,,

A short paper l a t e r will show

this.
Also solved by David Sowers and the proposer.

A FASCINATING RECURRENCE
H-37

Proposed by H.W. Gould, West Virginia University,

Find a triangle with sides

Morgantown, West. Va.

n + 1 , n, n - 1

having integral

area.

T h e f i r s t t w o e x a m p l e s a p p e a r t o be 3 , 4, 5 w i t h a r e a 6; a n d 1 3 , 1 4 ,
15 w i t h a r e a 8 4 .

1965

ADVANCED PROBLEMS AND SOLUTIONS

51

The p r o p o s e r ' s p a p e r c o m p r e h e n s i v e l y d i s c u s s i n g this p r o b l e m


will soon a p p e a r in the Q u a r t e r l y .
NO SOLUTIONS RECEIVED
H-38

Proposed by R.G. Buschman, SUNY,Buffalo,

(See F i b o n a c c i N u m b e r s ,

N.Y.

Chebyshev P o l y n o m i a l s ,

Generaliza-

tions and Difference Equations Vol. 1, No. 4, Dec. 1963, pp.


Show

+ (-b)r u

(u
n+r

H-40

)/u

= \

n- r
n
CORRECTED

Proposed by Walter Blumberg, New Hyde Park, L.I.,

Let U, V, A and B be i n t e g e r s ,
ditions

(i)

U > 1,

(ii)
(iv)

V - ^/(U

r
N.Y.

subject to the following con-

(U, 3) = 1;
2

1-7.)

(iii)

(A, V) = 1;

-l)/5

Show A U+BV is not a s q u a r e .


CONVOLUTIONS AND OPTICAL 2-STACK
H-39

Proposed by Verner E. Hoggatt, Jr., San Jose State College, San Jose,

California

Solve the difference equation in closed form


C

n+2

= C

n+1

+ C

+ F

n+2

'

where

C, = 1, C 0 = 2, and F
is the nth F i b o n a c c i n u m b e r .
1
Z
n
two s e p a r a t e c h a r a c t e r i z a t i o n s of t h e s e n u m b e r s .
Solution by L. Carlitz, Duke University,

Since

Durham, N.C.

C 2 = C, + C 0 + F - we have a l s o C = 0.

c(t) = x c n t n

then it follows from


n+2

n+1

n+2

that
(l-t-t2)C(t) = 1
0

F n t n = *
1-t-t

Thus
(1)

C(t) =

y
2

(l-t-r)
Expanding we get

If we put

Give

52

ADVANCED PROBLEMS AND SOLUTIONS


00

C(t) = t

2 (r+l)(t+t 2 )
r=0

= 2 (r + l ) t r + 1
r=0

February

(l)tS

2
s=0

n
= 2

n+1

n=0
so that

(r+l)(

n-r

r=0

n
r
C ,, = 2 (r+l)(
) = 2
)(n~r)
/x
v( n - r + l /x
n+1
n-r'
r '
r=0
2r<n

Another explicit e x p r e s s i o n that follows from (1) is


n-1
C

, = 2 F F
n-1
r n-r
r=l
Next is we differentiate
t

=2
i-t-t "
2

we get
1+t

(1-t-t )
which yields

F tn
n

2 (n+l)F^^
tn
2 " ^ x
'n+1
0

C n + Cn - 27 = (n+l)F n+1
,.

A consequence of this is
C

(-l)k(n-2k+l)F

, , ,,
n-2k+l

2k < n
F i n a l l y c o n s i d e r the n u m b e r
C = A n F + BnL
n
n
n
We find that
C 1n+2
, 9 - C'n+1. - Cn = A(F n+2
X 9 + F ) + B(L , 9 + L )
n
n+2
n
Since

L , 9 + L = 5(F
-F )
n+2
n
n+2
n

we get
C'_i_o
, 9 + v(A-5B)Fn
n+2 " C'n+1, - C n = v(A+5B)F
' n+2

1965

ADVANCED PROBLEMS AND SOLUTIONS

Hence for

A = 6, B = 1/10

it follows that

n+2

Clearly
C

53

n+1

= n/2 F

Taking n = 0 we get b = 0o

n+2

+ n/10 L + a F + bL
'
n
n
n
For

n = 1 we get a = 2 / 3 0

Therefore

we have
n L
C = n / 2 F + n / l O L + 2/5 F =
n
'
n
'
n
n

+ 2F
li
5

Also solved by Ronald Weimshenk, John L. Brown, Jr., Donald Knuth, H.H. Ferns and the proposer.

E d i t o r i a l Note:

Another c h a r a c t e r i z a t i o n , b e s i d e s the convolu-

tion
31+1

(n+l)L , 9 + 2 F ,_

C ,. = 1 F F
n+1

Jl+5

r n-r

2+i

r=l
i s the n u m b e r of c r o s s i n g s of the i n t e r f a c e , in the optical s t a c k in p r o b lem B - 6 , Dec. 1963, p . 75, for all r a y s which a r e r e f l e c t e d
If
fQ(x) = 0, f x (x) = 1, and f n + 2 (x) = xf n + 1 (x) + y x )
the F i b o n a c c i p olynomials, then
f (1) = F
and f ' ( l ) = C ,
n
n
n
n-1

n-times,
,

XXXXXXXXXXXXXXX

MATH MORALS
B r o t h e r IL Alfred
A tutor who t u t o r e d two r a b b i t s ,
Was intent on r e f o r m i n g t h e i r habits 0
Said the two to the tutor,
"There are rabbits much cuter,
But n o n - F i b o n a c c i , d a g n a b i t s . " '
The author has j u s t taken out poetic l i c e n s e # F q

a c c o r d i n g to one

c l a u s e of which it is p e r m i s s i b l e to c o r r u p t c o r r u p t e d w o r d s ,
XXXXXXXXXXXXXXX

PRODUCTS OF ODDS
SHERYL B. TADLOCK*
Madison College, Harrisonburg,Virginia

The following are used in the proofs of the identities:


3n

F =
n

- a

n
= Pn + an. ( N

, L

p - a

=(-Dn,

where
1 +/5
F

F
2k+l

=F
2j+l

1 - , 5

+ F
k+j+1

k-j

pk+j+l

2
2
F , , . , . + F. .
k+j+1
k-j

and a

_ Qk+j+l

>

Pk-j

- a k "J'

T^cT

~r^r

P2k+2j+2 +a2k+2j+2 - 2(Pa)k+J+1-2(Pa)k-Vk-2J+a2k-2J

2k+l /Q 2j+l
-2j-l. , 2k+l, 2j+K - 2J j - l ,
P
(P J
+ P J ) + a
(a
+a
)

2(pa) k+j (Pa + (Pa)- 2j )


(P-

a) 2

2j+l
2j+l
lx-2j-l
1) J a J
= -a J
a-2j-l=(.ir2j-lp2j+l
= _p2j+l

Recalling that

and

2j-l

and that the last term has the value of 0, the above expression becomes
F2
k+j+1

F2
k-j

P 2 k + i ( p 2 j + 1 - a2j+1) - a 2 k + 1 ( p 2 j + 1 - a2j+1)
(P . Q ) '
_ ( p 2 k + 1 - a 2 k + 1 ) ( p 2 J + 1 - a2J+1)
(P-

(P - a)

a)

But the right-hand side is of the form


F

k+j+l

+ F

k-j

Student
54

?k+1F2'+1'

2k+l F 2j+l

Therefore,

1965
2

PRODUCTS OF ODDS

2k+lL2j+l

k+j+l " L k~j

m2k+l ^
= (P
+ a

2k+lL2j+l

55

4( l)

"

2k+l W A 2j+l ^ 2j+l


+ a
>(P
>

= p 2k+2j+2 + a 2k+2j+2 + p 2k+l Q 2j+l

=
Observing that

Pk
a

2k+2

aJ

J+Z +a2k+2J+Z
,
= (-1)J

k-iJ 0 Ji-k
p

pk

, ,J-k

= (-1)J

and adding and s u b t r a c t i n g

pk

J+1ak+J+1(pk-JJ+k

= (-1)J

^k+l^j+1

k+

k-iJ k-iJ

2p

+p

. , ,j-k

= (-1)J

pZ
a

ak^pJ-k)

^J

2k-2i
J

the above e x p r e s s i o n b e -

comes
L

2k+1L2J+1 = P 2 k + 2 J + 2 + a2k+2J+2+2Pk+J+1ak+J+1

-ZP^J-^^^

+ (Pa) k + J + 1 ( ( - l ) J - k p 2 k " 2 J + ( - l ) J " k a 2 k - 2 j )


p 2k+2j+2 + 2 p k + j + l a k+j+l + Q 2k+2j+2
+ (Pa) k + J + 1 ( - 2 + ( - l ) J - k P 2 k - 2 J + ( - l ) J - k a 2 k - 2 J )

= (pk+J"1+ak+j +

)2+(-l)k+J+1(-l)J"k(p2k"2J+a2k-2j-2(-l)k-J)

= L k+j + l + ( - D 2 J + 1 ( P 2 k _ 2 J + 2 p k " J a k " J + a 2 k " 2 J - 2 ( 3 k - j a k - j )


Noting that

.2t.1)k-j(_1)2j+l

_2(_x

- 2(-l)k"J(-l)2j+1
} k - j ( _ 1} = 2 ( _ 1 } k - j

we have
L

2k + l L 2 j + l = L k + j + l - K "

2
T2
^j+l-^-i

+ ak j 2

" )

, .,.nk-j
^

+ Z

" 2<^)k"j]

+ 2(_1)k-j
J + Z

Therefore,

2k+lL2j+l=Lk+j+l-Lk-j+4(-1)k"J

+ 2

(- 1 > k " J

56

PRODUCTS OF ODDS
By using the identity

February

- 5 F 2 = 4 ( - l ) n (Vol. 1, No. 1, p. 66,

\}

this Quarterly), it can easily be shown that


Li2 . - I " L? . + 4v( - l ) k ~ j = L?, .,_ - 5F? . = 5F? , . , , - L? . .
k+j+1
k-j
k+j+1
k-j
k+j+1
k-j
Thus, we have proofs of the following Fibonacci identity and the analogous
Lucas identities for products of odds:
(1)
v ;

F
F
= F2
+ F2
*2k+l 2j+l
k+j+1
k-j

<2>

L2k+1L2.+1 =

(3)

2k+l L 2j+l

= L

(4)

2k+l L 2j+l

= 5F

I^+j+1-I^_j+4(-l)k-J
2
k+j+l "

5F

2
k+j+l "

2
k-j
2
k-j

'

These four identities correspond closely to those given for products of evens in this Quarterly, Vol. 2, No. 1, p0 78.

xxxxxxxxxxxxxxx

NOTICE TO ALL SUBSCRIBERS] i i


Please notify the Managing Editor AT ONCE of any address change.
The Post Office Department, rather than forwarding magazines mailed
third class, sends them directly to the dead-letter office.

Unless the

addressee specifically requests the Fibonacci Quarterly be forwarded


at first class rates to the new address, he will not receive it.

(This

will usually cost about 30 cents for first-class postage. ) If possible,


please notify us AT LEAST THREE WEEKS PRIOR to publication dates.
February 15, April 15, October 15, and December 15.

EXPLORING SCALENE FIBONACCI POLYGONS


Proposed by BROTHER U. ALFRED on page 60, October 1963
"The Fibonacci Quarterly"
C.B.A. Peck

The sequence of Fibonacci numbers may be defined by


(1)

F n = 0, F, = 1, and F = F . + F . for n 2 2.
0
1
n
n-1
n-Z

We describe a way of deciding when a set S of m distinct numbers


drawn from the sequence

F?i

of some plane polygon with m

F, F . , . . .
sides.

corresponds to the sides

If they do we call

S a (scalene

Fibonacci) polygon, for short.


To prove the result,

we find it convenient to use the following

identities, easily proved from (1) by induction on k and n, respectively:


k
(2)
x

=F

01
n-2k

r n
n-2i+l

for

n^4

and 0 < 2k C n,

i=l
n-2
(3)

F >

F. for

n ^ 1 (the sum is zero if n = 1, 2, 3)

i=2
Suppose once and for all that

is the largest number in S.

If

we denote by S(n, k) the set of numbers appearing in (2), then S is a


polygon if and only if it properly contains some

S(n, k).

If it equals

S(n, k) for some k we call it a degenerate polygon.


Proof:

If F

^ S, then by (2) S contains no S(n, k).

By (3)

exceeds the sum of the other numbers in S, which shows that


even a degenerate polygon. Now suppose that

S is not

, S (so that n 2. 3)

and proceed downward through the sequence in (1), starting with


and stopping short of

F, .

The numbers alternate in and out of S until

one of two things happens.


1.

S is found to containno

S(n, k), either because the alterna-

tion stops at an adjacent pair not in

S,

say

?.,

. ,

with

n-2j-l 2. 2, or continues to the bottom (here we set n-2j-l = 1 or 0 according as

n is even or odd). Then every number in S other than

occurs in either
57

58

EXPLORING SCALENE FIBONACCI POLYGONS


n-2j-2

February

V
F . or X F - . , , .
^
i
^
n-2i+l
i=2
i=l
The f i r s t sum < F - . by7 (3), whence the sum of e a c h < F
n-2j
n
Thus S is again not even a d e g e r n a t e polygon.
2.
F

-,,

by
7 (2).

The a l t e r n a t i o n stops with an adjacent pair in S, say F

with n - 2 k ^ 2 ,

so that S(n, k) is in S.

?,

,,

Then (2) shows that

S is a (degenerate) polygon if t h e r e a r e (no) n u m b e r s in S b e s i d e s


those in S(n, k), on the grounds that F

(4) the sum of the other n u m -

b e r s in S.
Could two s e t s of n u m b e r s drawn from

F ? , F^, F . , . . .

p o r t i o n a l to the lengths of the sides of a single polygon?


possible,

be p r o -

This is not

at any r a t e , when the n u m b e r s in each set a r e distinct, for

suppose that we did have two s c a l e n e Fibonacci polygons with l a r g e s t


sides

each other,

and F KT (N > n) p r o p o r t i o n a l to a third polygon, hence to


say in the r a t i o P > 1.

We have just seen that if F

the l a r g e s t n u m b e r in such a set, then F

is

m u s t belong to it.

Since

the l a r g e s t and second l a r g e s t sides c o r r e s p o n d , we have P F

= FM

and P F

, = F XT , . By
= F AT 0, and n-2
J (1), we have, then, P F
n-1
N-l
n - 20
N-2
f u r t h e r applications of (1) yield finally P F n = F
. By (1), the l . h . s.
is z e r o and the r . h . s.

positive, which is a b s u r d .

An i n t e r e s t i n g e x e r c i s e is to u s e this a r g u m e n t (with suitable


amplification of the last sentence) on any two Fibonacci polygons such
that in at l e a s t one of them t h e r e a r e n u m b e r s whose s u b s c r i p t s differ
by only one or two. We need something s t r o n g e r for such polygons as
n

n-3
n-3
n-3, n-3
n-3
The g e n e r a l i z a t i o n of (2) which s e e m s to be called for is some
c h a r a c t e r i z a t i o n of the coefficients in inequalities of the form

Fn i 2 ^

a.1 F.1

i=2

w h e r e the a . ' s a r e nonnegative i n t e g e r s .


XXXXXXXXXXXXXXX

NOTE ON THIRD ORDER DETERMINANTS


BROTHER U. ALFRED
St. Mary's College, California

The r e c e n t exhaustive investigation of nine-digit d e t e r m i n a n t s


by Bicknell and Hoggatt that a p p e a r e d in the M a t h e m a t i c s Magazine of
M a y - J u n e , 1963, r a i s e s an i n t e r e s t i n g question [ l ] .

Given that

9 4 2
3 8 6
5 1 7
or any equivalent a r r a n g e m e n t producing the s a m e set of p r o d u c t s
h a s a m a x i m u m value of 412, would we obtain . a m a x i m u m for any
other nine consecutive positive i n t e g e r s using the s a m e r e l a t i v e a r rangement?

This note will offer a negative a n s w e r and indicate the

m a x i m u m for all positive v a l u e s .


First,

a s m a l l amount of t h e o r y is in o r d e r .

d e t e r m i n a n t has e l e m e n t s

a.

If a t h i r d o r d e r

and a fixed quantity b is added to each

e l e m e n t the r e s u l t i n g d e t e r m i n a n t would be:


a

z +

a4 + b

a_ + b

a? + b

+b

a3+b
a

6 +

a9 + b

Subtract the second column from the third and the first from the second to obtain
a

+b

a4 + b

~5

a? + b

a 0 - a_

"4
a

" a

from which it is evident that the value of the a l t e r e d d e t e r m i n a n t is


D + Ab ,
w h e r e D is the value of the original d e t e r m i n a n t and A is the
of the t h r e e m i n o r s formed from the second and t h i r d c o l u m n s .
panded and grouped a p p r o p r i a t e l y we obtain
59

sum
Ex-

60

NOTE ON THIRD ORDER DETERMINANTS

February

A = (a][a5+a5a9+a9a1) + ( a ^ ^ + a ^ + a ^ ^ + (a3a4+a4ag+a8a3)

- (a1a6+a6a8+aga1) - ( a ^ + a ^ + a ^ ) - ( a ^ + a ^ + a ^ )
This coefficient

X gives the change in the value of the d e t e r m i n a n t

a s we add 1 to each of its e l e m e n t s .

See [2] for another u s e .

It should be noted that the groups in

A a r e the s a m e as those

for the positive and negative t e r m s of the d e t e r m i n a n t expansion and


hence any a l t e r a t i o n of the a r r a n g e m e n t of d e t e r m i n a n t e l e m e n t s which
leaves the expansion unchanged will a l s o be without effect on A .
An independent investigation shows that the m a x i m u m value of
A is 81 when the e l e m e n t s of the d e t e r m i n a n t a r e the nine digits, while
the value of
80.

A for the d e t e r m i n a n t giving a m a x i m u m of 412 is only

Thus, the s m a l l e r valued d e t e r m i n a n t with A = 81 will eventually

o v e r t a k e the l a r g e r as the e l e m e n t s of the d e t e r m i n a n t s a r e i n c r e a s e d


uniformly.
By calculating

A for the l a r g e s t values given in the table of

B i c k n e l l a n d H o g g a t t ( R e f . 1, p . 152)Ais found to be 81 for 405 = 630-225a


and 630-225c.

Adding n to each e l e m e n t of 630-225a, for e x a m p l e ,

will produce a d e t e r m i n a n t of value 405 + 81 n; doing likewise for the


original m a x i m u m
412 + 80n.

determinant

of value 412 p r o d u c e s a value of

To find when t h e s e will be equal, set


405 + 81n = 412 + 80n

the solution being n = 7.


Thus,

if we have nine consecutive positive i n t e g e r s beginning

with m, the m a x i m u m value that can be achieved is 412 + 80m if


m

iL 8; the m a x i m u m p o s s i b l e is 405 + 81m if m ^ 8.

REFERENCES
1,

2.

M a r j o r i e Bicknell and V e r n e r E. Hoggatt, J r . , "An Investigation


of Nine-Digit D e t e r m i n a n t s , " M a t h e m a t i c s Magazine, 36(1963),
147-152.
M a r j o r i e Bicknell and V e r n e r E. Hoggatt, J r . , " F i b o n a c c i Mat r i c e s and Lambda F u n c t i o n s , " The Fibonacci Quarterly,- 1(1963)
April, pp. 4 7 - 5 2 .
XXXXXXXXXXXXXXX

MORE ON FIBONACCI NSM


JEREMY C. POND and DONALD F. HOWELLS*
Sussex, England and San Jose State College, San Jose, California

Fibonacci Nim [1] was o r i g i n a l l y stated as follows:


"Consider a game involving two p l a y e r s in which initially t h e r e
is a group of 100 or l e s s o b j e c t s . The f i r s t p l a y e r m a y r e d u c e
the pile by any Fibonacci n u m b e r ( m e m b e r of the s e r i e s 1, 1, 2,
3, 5, 8, 13, 21, . . . ). The second p l a y e r does likewise. The
p l a y e r who m a k e s the last move wins the g a m e . "
Let p e r s o n s A and B be playing the game which A w i n s .

If A is

to win he m u s t be able to r e d u c e the pile to z e r o on his final m o v e .


Thus A m u s t draw from

0+F

(n = 1, 2, 3, . . . )

on his final m o v e .

Looking at the sequence of the n u m b e r of objects from which A


m u s t draw to win on the final move, 1, 2, 3, 5, . . . , we see that 4 is
the f i r s t positive i n t e g e r m i s s i n g . If B is forced to play with 4 objects
r e m a i n i n g , A can c e r t a i n l y win the g a m e .
Now suppose A gets the opportunity to draw from
9, 12, . . ).

4+F (5, 6, 7,

A will be able to r e d u c e the pile to 4 objects and can

continue to win.
The s m a l l e s t positive i n t e g e r that is not contained in the union
of the s e t s

(1 0 + F ]) and {4+F ) is 10. If B is forced to d r a w from


n
A n'
a pile of 10 objects, B cannot reduce the pile to 4 or 0 but B will leave
A in a position to r e d u c e the pile to 4 or 0 and thus A can win.
Now we wish to g e n e r a t e the sequence of positions from which it
is unsafe to draw (0, 4, 10, . . . ).

Let U , = 0 . Then U^ is the s m a l l -

e s t positive i n t e g e r which is not equal to U, +F

(n = 2, 3, . . . ).

is the s m a l l e s t positive i n t e g e r which is not equal to U, +F

or

U~

U?+F

(n= 2, 3, . . . ) .
Therefore
U (r = 2, 3, . . ) is the s m a l l e s t positive i n t e g e r
w h i c h i s not equal to U.+F , w h e r e t = 1, 2, . . . , r - 1 and n = 2, 3, . . .
^
t n

Student
61

62

February

MORE ON FIBONACCI NIM

410
14
20
24
30
36
40
46
50
56
60
66
72
76
82
86
92
96

1
5
11
15
21
25
31
37
41
47
51
57
61
67
73
77
83
87
93
97

2
5"~
12
16
22
26
32
38
42
48
52
58
62
68
74
78
84
88
94
98

3
7
13
17
23
27
33
39
43
49
53
59
63
69
75
79
85
89
95
99

8
12
18
22
28
32
38
44
48
54
58
64
68
74
80
84
90
94
100

5
9
15
19
25
29
35
41
45
51
55
61
65
71
77
81
87
91
97

13
17
23
27
33
37
43
49
53
59
63
69
73
79
85
89
95
99

21
25
31
35
41
45
51
57
61
67
71
77
81
87
93
97

34
38
44
48
54
58
64
70
74
80
84
90
94
100

55
59
65
69
75
79
85
91
95

89
93
99

The first player can always win if he starts on some position not
equal to U

(r = 1, 2, ...) and always reduces the pile to some U .

Here are all the values of U


0
76
169
254
332
410
488
572

4
82
176
260
338
416
494
576

10
86
186
264
342
420
498

14
92
192
270
348
426
504

20
96
196
274
352
430
510

24
102
202
280
358
436
514

30
108
206
284
364
442
520

36
112
212
290
368
446
524

thus far computed:


40
118
218
296
374
452
530

46
122
222
300
378
456
534

50
128
228
306
384
462
540

56
132
232
310
388
468
552

60
138
238
316
394
472
556

66
150
242
322
400
478
562

72
160
248
326
406
484
566

4.
5.

The following observations can be made:


U r+1
., = Ur + some non-Fibonacci number.
If U 1 - U =4, then U ,,-U ,,9^4 since 4+4=8=F/.
r+1 r
r+2 r+1'
o
Thus the average difference of U ,,-U = 5 , r=l, 2, 3, . . .
to
r+1
r
The density of (U } in the positive i n t e g e r s m u s t be ^ l/5
The p r o b a b i l i t y that the s t a r t i n g p e r s o n can win is Z. 4 / 5 if noth-

10

ing is known about the s t a r t i n g position of the g a m e .


The following questions a r e left u n a n s w e r e d :
Is t h e r e a closed form solution for ] U r | ?

1.
20
3.

1965

M O R E ON F I B O N A C C I NIM

2.

W h a t i s t h e l i m i t i n g d e n s i t y of
Similar

results

a r e found

(U

when

63

} in the positive i n t e g e r s ?

one

considers

"Lucas

Nim"

a n a l o g o u s to F i b o n a c c i N i m .

REFERENCES
1.

B r o t h e r U. A l f r e d ,
onacci Quarterly,

2.

"Research Project:

Fibonacci Nim, " F i b -

1 ( 1 9 6 3 ) , N o . 1, p . 6 3 .

Michael J. Whinihan,
1 ( 1 9 6 3 ) , N o . 4, p p .

"Fibonacci Nim, "

Fibonacci

Quarterly,

9-13.

xxxxxxxxxxxxxxx
ON SUMS F 2

F2

BENJAMIN SHARPE
State University of New York at Buffalo

F o r m u l a s f o r t h e s u m of t h e s q u a r e s of F i b o n a c c i n u m b e r s a r e :
(1)

2
n+2k

2
n =

(2)

2
n+2k+l

(3)

2
n+2k "

(4)

2
n+2k+l '

+ F

2
n =

+ F

2
n =

n+2k-2
F

2
n =

2k+1

2k
F

n+2k+l

+ F

2k-1

2n+2k-l

2n+2k+l

2n+2k
F

n-1

n+2

+ F

2k

2n+2k+2

V a l i d i t y of t h e a b o v e i s e s t a b l i s h e d b y u s i n g :
,_,
F

1 . n
_
0n,
= - ( a
- P ), L
n
^5
For example:

= a

n , 0n
1 +J5
1 -J5
,
Q
a
v
v
+ P , a =
, p =
, a P = -1

5 v^ * , , , - F 2 ) =
n+2k+l
n'
(a2n+4k+2

+ p2n+4k+2)

2n+4k+2 "

5(F

n-l

n+2

(a2n + p2n}

2Qnpn (Q2k+lp2k+l

2n ^(-l)n(-2) = L 2 n + 4 k + 2 - ^
+ F

2k

2n+2k+2)

_1}

- (-l)11"1

, 2 n + l a 2 n + l x ^ , 2n+4k+2 __, a 2 n + 4 k + 2 x
i i - l ^ n - l , 3,A3N
(a
+ P
) + (a
+p
)- a
P
(a +P ) - a
L

2n+4k+2

2ka2k, 2n+2,ft2n+2.
p (a
+p
)=
+ (L

2n+l "

2n+2- ) " (~l)

XXXXXXXXXXXXXXX

2n+4k+2 "

2iT

(-1)

3e

PHYLLOTAXIS
EJ.KARCHMAR
Control Data, Palo Alto, California

Leaves a r e commonly a r r a n g e d on the plant s t e m a c c o r d i n g to a


pattern.

If the p a t t e r n is "whorled, " s e v e r a l leaves a r i s e from the

s a m e node, at i n t e r v a l s along the s t e m .


the a r r a n g e m e n t is t w o - r a n k e d .

If the p a t t e r n is "distichous, "

However, the m o s t common p a t t e r n

of a r r a n g e m e n t is " s p i r a l . "
The m o s t a c c u r a t e method for studying plant phyllotaxis is by
t r a n s e c t i n g the apical bud and making o b s e r v a t i o n s on the c r o s s - s e c tion.

When one e x a m i n e s such a c r o s s - s e c t i o n ,

the m o s t striking

feature to m e e t the eye is the s p i r a l a p p e a r a n c e of the a r r a n g e m e n t of


leaf p r i m o r d i a .

It has been found that t h e r e is a definite,

s p i r a l a p p e a r a n c e of the a r r a n g e m e n t of leaf p r i m o r d i a .
found that t h e r e is a definite,

heritable
It h a s been

h e r i t a b l e s p i r a l a r r a n g e m e n t which can

be designated (in m o s t c a s e s ) by two n u m b e r s : the n u m b e r of s p i r a l s


which t u r n in one direction,

and the n u m b e r which t u r n in the other

(these c u r v e s a r e called " p a r a s t i c h i e s " ) .

The i n t e r s e c t i o n s of t h e s e

two s p i r a l s y s t e m s delineate " q u a s i - s q u a r e s , " within which a r e found


the leaf p r i m o r d i a (2, 4, 40).
In an overwhelming n u m b e r of s p e c i e s (434 s p e c i e s in the Angios p e r m a e and 44 s p e c i e s in the G y m n o s p e r m a e w e r e found by T. Fujita
in 1938) the p a r a s t i c h y n u m b e r s fall in the Fibonacci Sequence, the
m o s t c o m m o n p a i r s of n u m b e r s being 2:3 and 3:5 (see Appendix) (40).
When the p a r a s t i c h y n u m b e r s do not fall in the Fibonacci Sequence,
they r e g u l a r l y fall into one of the other s u m m a t i o n s e r i e s (see Appendix,
footnote).
It has a l s o beenfound by i n v e s t i g a t o r s in the field (2, 14, 40) t h a t
the angle between adjacent leaf p r i m o r d i a i s , in a convincing n u m b e r
of c a s e s , a p p r o x i m a t e l y 137 3 0 ' . This is v a r i o u s l y called the "ideal
angle, " the "divergence angle, " and the " L i m i t d i v e r g e n z . " This angle
can be obtained m a t h e m a t i c a l l y by applying the limiting value of the
Fibonacci Sequence u / u , , :
^
n' n+1
360 - (0.6180)(360) = 13730' o
64

965

PHYLLOTAXIS

65

PhyLlotaxis has been a field of i n t e r e s t for c e n t u r i e s .

Since

1900 s e v e r a l t h e o r i e s have been offered as explanation of some of the


phenomena of phyliotaxis 0

Some e x p e r i m e n t a t i o n has been done to d e -

t e r m i n e the effect of e n v i r o n m e n t or m e c h a n i c a l damage on phyllotaxis


(6, 11, 21, 25, 42); and some X - r a y and c h e m i c a l effects on the d e velopment of leaf a r r a n g e m e n t have been noted (17, 20, 22, 23).
ever,

How-

after 1920 v e r y little has been published on this subject; p e r -

haps the feeling is that t h e r e a r e so m a n y m o r e fruitful and l e s s


" m y s t e r i o u s " a r e a s of botanical i n t e r e s t ,
alone 0

that this one is best left

Also, the subject s e e m s to lie m o r e p r o p e r l y in the r e a l m of

biophysics, which is a r e l a t i v e l y new field*


The s p i r a l a r r a n g e m e n t d i s c u s s e d in (1) above is not p e c u l i a r to
plants.

It is a l s o found in the shells of f o r a m i n i f e r a (4), nautili, and

other a n i m a l s .

It is the opinion of Church (4, p . 48) that the factor

c o m m o n to both plants and the f o r a m i n i f e r a is "the building of new units


one at a t i m e , and it thus a p p e a r s that this is the e s s e n t i a l factor
behind all such p r e s e n t a t i o n of Fibonacci r e l a t i o n s , to all t i m e . "
Church a l s o feels that the Fibonacci phyllotaxis is phylogenetically p r i m i t i v e (4, p . 13).
" . . . v e r y a d m i r a b l e s p i r a l a r r a n g e m e n t s , in which Fibonacci
s y m m e t r y m a y be d i s t i n c t l y t r a c e d , obtain in the c a s e of m a n y
of the m o r e m a s s i v e Brown Seaweeds ( P h a e o p h y c e a e - F u c o i d e a e ) ,
in the o r i e n t a t i o n of the m o r e or l e s s frondose or leaf-like late r a l r a m u l i ; leaving little doubt that the p h y l l o t a x i s - m e c h a n i s m
i s , in fa-ct, a still older function of the axis of m a r i n e types of
vegetation, and that the p r e s e n t a t i o n of such phenomena, even in
a m o r e e l a b o r a t e d and s p e c i a l form, can be but the continuation
and amplification of f a c t o r s of m a r i n e phytobenthon; and that it is
to the sea that one m u s t look for the origin and p r i m a r y intention
of this r e m a r k a b l e r e l a t i o n . " (4, ppQ 37-38)
T h e r e s e e m s to be little doubt that the p r i m a r y m e c h a n i s m r e sponsible for Fibonacci phyllotaxy is genetic in nature., r a t h e r than being a function of growth conditions such as availability of,
for, illumination.

and need

In the w o r d s of Church,

"It can only be concluded that the plant is somehow biased from
the f i r s t i n f a v o u r of m e m b e r s a r r a n g e d one by one in a F i b o n a c c i
sequence; and the suggestion i m m e d i a t e l y offers that this m a y
be in some way the e x p r e s s i o n of the i n h e r i t a n c e of the equipm e n t of a p r e c e d i n g p h a s e and the solution of a m u c h older p r o b l e m . " (4, p e 53)

66

PHYLLOTAXIS

February

I suggest that, by a c o n s i d e r a t i o n of a type of o r d e r and s y m m e t r y


so basic to living m a t t e r ,

one m a y p e r h a p s gain some insight into the

p r o b l e m of the origin of that o r d e r ,


E d i t o r i a l Comment:

A m i m e o g r a p h e d 46 e n t r y annotated bibliography

is available on r e q u e s t f r o m the Fibonacci A s s o c i a t i o n .


d i r e c t l y to B r o t h e r U. Alfred,

Send r e q u e s t s

St. M a r y ' s College, California 94575.

XXXXXXXXXXXXXXX

CORRECTIONS:

Volume 2, Number 3

In the poem "A Digit M u s e s " by B r o t h e r U. Alfred, page 210, we wish


to say "oh pshaw, no phi! " since PHI was omitted from the end of the
sixth line.
Page 204: The symbol
displayed equation.

CORRECTIONS:

cf> was omitted from the n u m e r a t o r of the last

The n u m e r a t o r i s , of c o u r s e ,

0(x).

Volume 2, Number 4

P a g e 290: Title should have

rr in the blank after the second word in

the title and the eighth displayed equation should have a + and a - r e s p e c t i v e l y in the blanks between the second and t h i r d t e r m s and the
t h i r d and fourth t e r m s .
Page 281: Missing symbol in the f i r s t displayed equation i s , of c o u r s e ,
a s u m m a t i o n symbol.
XXXXXXXXXXXXXXX

FIBONACCI FANTASY: THE SQUARE ROOT OF THE Q MATRIX


MARJORIE BICKNELL
Adrian Wilcox High School, Santa Clara, California

The m a t r i x
1

h a s m a n y well known fascinating p r o p e r t i e s , one being that

o
where

n+1

F
l_ n

n-1.

is the nth Fibonacci n u m b e r . The Q m a t r i x a l s o h a s a


n
Fibonacci s q u a r e root, which can be exhibited after making a simple

definition.
We extend the r e l a t i o n s h i p s

Fk

= ( a k - /3 k )/( a - ) ,

a = (1 + / 5 ) / 2 ,

|3 = (1 -

t o a l l o w k to equal a n y i n t e g r a l m u i t i p l e of one-half.
m u l t i p l e s of one-half for a moment,
.2

f5)/2,

Considering odd

it is e a s y a l g e b r a i c a l l y to obtain

rT

(2n+l)/2 = [ L 2n + 1

, o-./

iv'n+l-i

'J/*

and
L

^2n+l)/2

2n+1

d i r e c t l y from the extended definition.

rr,

^'V*'

So then, all Fibonacci or Lucas

n u m b e r s whose s u b s c r i p t s a r e odd m u l t i p l e s of one-half a r e


Also, combining the two equations d i r e c t l y above yields

2
2
(2n+l)/2 " 5F(2n+l)/2
67

4i(-1)

complex.

68

FIBONACCI FANTASY:

THE SQUARE

February

Returning to the Q matrix, a square root of Q is given by [I]


"F /
3/2

Q,V2 _

L F

F /
1/2
F

1/2

-l/2-l

for, by applying the extended definition and simplifying,


F

3/2+Fl/2

and

= 1

3/2 F l/2

'

+ F

?/2+F?i/2 = 0

l/2 F -l/2 "

As suggested by the second of these equalities, we can write


F

-l/2

iF

l/2

By taking the determinant of the square root of Q,


1

2
3/2F-l/2 " Fl/2

'

Also, that
F
Qn/2

(n+2)/2

/?

matrices, from

n/2

(n-2)/2-

n/2

can be established by induction,


gebraic manipulation.

~]

using the extended definition and al-

By equating corresponding elements of equal

(Q ' ) = Q , we obtain

* (n+2)/2

n/2 "

n+1

and
F

(n+2)/2 F n/2

Taking the determinant of Q


F

+ F

n/2F(n-2)/2

yields

^
(n+2)/2 F (n-2)/2 " F n / 2

i.l,n/2

The reader can easily establish that

1965

ROOT OF THE Q MATRIX


F

= F
(2n+3)/2

(2n+3)/2

(n+3)/2 (n+2)/2

""

(2n+l)/2L(2n+l)/2

(n+3)/2Fn/2

2n+l
F

/ F
+ F F ,
3/2 n+1
n 1/2

(2n+l)/2

+ F

69

'
F

(n+l)/2Fn/2
'

(n+1 ) / 2 F ( n - 2 ) / 2

Let us p u r s u e a m o r e g e n e r a l r e s u l t .

'

It can be e s t a b l i s h e d by

induction that
Qp /

"F
(p+r)/r

F
p/r

F
L
L- p // r

(p-r)/rJ

r /

if we further extend the definition of Fibonacci n u m b e r s to include s u b s c r i p t s which a r e r a t i o n a l n u m b e r s .


1}
(-1)

Taking the d e t e r m i n a n t yields


.2

p/r

(p+r)/r

(p-r)/r "

p/r

As an e x a m p l e , since
Q

p/r

/P

Q(P

2 2 r
+r ) / P ?

c o n s i d e r a t i o n of the e l e m e n t s of t h e s e m a t r i c e s leads to

V + r 2 + r p ,/rp

= F(P+r)//(r+P)/P +

^/^^P

'

which is a g e n e r a l c a s e of the f a m i l i a r identity


F

= F
m+n+1

In g e n e r a l ,

F
+F F
m+1 n+1
m n

it s e e m s that identities which hold for i n t e g r a l sub-

s c r i p t s a l s o hold for our s p e c i a l i z e d r a t i o n a l s u b s c r i p t s .


Fibonacci s u b s c r i p t w e r e a complex n u m b e r ?

What if the

J. C. A m s o n [2] h a s

a n s w e r e d this question while d e m o n s t r a t i n g an analogy to the f a m i l i a r


c i r c u l a r and hyperbolic functions.

70

FIBONACCI FANTASY:

THE SQUARE

February

Am son defined modified Lucas functions as


luc z = (w

- w )/2A,

coLuc z = (w

+ w )/2,

where

z is a complex n u m b e r and w and w a r e the r o o t s of the


2
2
2
q u a d r a t i c equation x = Px - Q, with d i s c r i m i n a n t A = P - 4Q.
(Notice that luc z = ( F j / 2 , coluc z = (L )/2 when P = 1, Q = - 1 . )
A l g e b r a i c a l l y , we see that, among other i d e n t i t i e s ,
Q luc(-z) = - luc z
Q coluc(-z)

= coluc z

luc 0 = 0, coluc 0 = 1
luc 2z = 2 luc z coluc z
luc(z
Q

luc z

coluc z

+ coluc z. luc z ?

iuc(z 1 - z) = luc z, coluc z ? - coluc z, luc z ?

coluc(z
z
Q

+ z?)

2
+ z~) = coluc z, coluc z ? + A luc z ] luc z^

2
coluc(z, - z ? ) = coluc z. coluc z? - A luc z, luc z ?

2
2
2
coluc z + A luc z = coluc 2z
coluc z - A luc z = Q
(coluc z + A luc z)

= coluc nz + A luc nz.

C o m p a r i s o n of t h e s e Lucas functions with those d e r i v e d from the


c i r c u l a r functions defined by
, iz
-iz x / o .
/ iz , -iz x / o
s m z = (e
- e
)/2i,
cos z = (e + e
)/2
or those from the hyperbolic functions defined s i m i l a r l y by
sinh z = (e
r e v e a l s a close analogy.
equation is x

- e

)/2 ,

cosh z = (e

+e

)/2

Also, in the s p e c i a l c a s e that the q u a d r a t i c

= x + 1, we see a f a m i l i a r list of F i b o n a c c i identities

1965

71

ROOT OF THE Q MATRIX

e m e r g i n g for complex s u b s c r i p t s .

This fine r e f e r e n c e [2] was brought

to our attention by Prof. T y r e A. Newton.

REFERENCES
1.

The s q u a r e root of Q was suggested by Maxey Brooke in a l e t t e r .

2.

J. C. Amson,

" L u c a s Functions, " E u r e k a :

The J o u r n a l of the

A r c h i m e d e a n s , (Cambridge University), No. 26, October, 1963,


pp. 2 1 - 2 5 .
3.

S. L. Basin and V e r n e r E. Hoggatt, J r . ,

"A P r i m e r on the F i b -

onacci Sequence, P a r t II, " Fibonacci Q u a r t e r l y , 1:2, pp. 6 1 - 6 8 .

xxxxxxxxxxxxxxx
LETTER TO THE EDITOR
P. NAOR
The University of North Carolina
Chapel H i l l , N.C.

I r e a d with g r e a t i n t e r e s t your r e c e n t p a p e r "On the O r d e r i n g of


the Fibonacci S e q u e n c e s . " The g e n e r a l idea underlying your o r d e r i n g
p r o c e d u r e is excellent, but the r e p r e s e n t a t i o n can be i m p r o v e d and
(possibly obscure) r e l a t i o n s h i p s m a y be brought to light.
Consider (for the t i m e being) sequences for which D ^ 1 1 . F o r
r e a s o n s which will soon become c l e a r I p r e f e r to define the n u m b e r f
(in your notation) a s the f i r s t t e r m in the sequence, 0 , say. You c o r r e c t l y pointed out that "a negative sequence m a y be obtained from a
positive sequence by changing the signs of all t e r m s " . . . ; however,
t h e r e is another ( r a t h e r simple) o p e r a t i o n which e s t a b l i s h e s an .equivalence between two s e q u e n c e s . Consider a sequence

.
and a s s u m e ,

<_4> 0_3> < 2 j


for c o n v e n i e n c e ,

0_1> 0O> 0 1 > 0 2 > <py 0 4 >


that the monotonic p o r t i o n is p o s i t i v e .

It i s e a s y t o v e r i f y t h a t 0
i s p o s i t i v e ( n e g a t i v e ) if n i s e v e n (odd)
w h e r e n is a n o n - n e g a t i v e i n t e g e r . Next view an a s s o c i a t e d sequence
{ 0' }
defined by
0'
= 0
if n i s e v e n
n
^n
d>\ = <6_
if n i s o d d .
< n
r' + n
It i s e l e m e n t a r y t o s h o w t h a t { 0 ' } i s a F i b o n a c c i s e q u e n c e ( w i t h
the monotonic p a r t positive) - thus Fibonacci sequences typically a p p e a r

72

L E T T E R TO THE EDITOR

February

in p a i r s - { (f>n } being { 0 } - although the p o s s i b i l i t y of a sequence


being s e If - a s s o c iate d (a nd thus a p p e a r i n g to be single) cannot be ruled
out a p r i o r i . Now for a series"To"Be~"self-associated in the following
m u s t hold - <f> - + tp
so that the c e n t r a l t e r m 0
becomes

and, if we a r e i n t e r e s t e d only in s e q u e n c e s which a r e not i n t e g r a l m u l tiples of other s e q u e n c e s , it b e c o m e s c l e a r that one, and only one,
such sequence e x i s t s , to wit
.......

- 4 , 3, - 1 , 2, 1, 3, 4,

.....

whose D equals 5. Let this sequence be denoted a s the o r d i n a r y selfa s s o c i a t e d sequence,, However, t h e r e e x i s t s in addition"an exTra_ordin a r y s e l f - a s s o c i a t e d s e q u e n c e . If we a d m i t (which we did not before)
tEe~possibility <f> = 0, we have

and the only "prime 1 1 solution is the Fibonacci sequence


......

- 3 , 2, - 1 , 1, 0, 1, 1, 2, 3,

.....

You note, of c o u r s e , that in this single c a s e f equals <f> and not


4>i 0 This sequence is indeed e x t r a o r d i n a r y in s e v e r a l r e s p e c t s : In
c o n t r a d i s t i n c t i o n to all other s e q u e n c e s it h a s the p r o p e r t y that (f> is
positive (negative) if n is odd (even). Also <p < ] <f> , J is t r u e in
this c a s e w h e r e a s in all other c a s e s the inequality nolds in the opposite
d i r e c t i o n . An exceptional behavior of D will be d i s c u s s e d in this l e t t e r .
It is then m y p r o p o s a l to c h a r a c t e r i z e the F i b o n a c c i s e q u e n c e s
not by (f , f, ) but r a t h e r by ( 0 1 , <f>, ).
This r e p r e s e n t a t i o n h a s
n u m e r o u s a d v a n t a g e s : The two mutually dual F i b o n a c c i s e q u e n c e s m a y
be r e p r e s e n t e d by _one p a i r of b r a c k e t s , e. gD , what you r e p r e s e n t a s
(1, 4) and (2, 5) would become in m y notation (-2, 1) and ( - 1 , 2); both in
one r e p r e s e n t a t i o n would be w r i t t e n as [2, l] with the a g r e e m e n t that
the l a r g e r (in absolute value) n u m b e r p r e c e d e s the s m a l l e r n u m b e r .
The o r d i n a r y s e l f - a s s o c i a t e d (or self-dual) sequence would be [ 1 , 1 ]
w h e r e a s the e x t r a o r d i n a r y s e l f - d u a l sequence d e s e r v e s s p e c i a l notation, e. g. ( 1 , 1 ) .
Consider now the quantity D as defined in your p a p e r
?

D = ff - f. f - r
In t e r m s of

<j> and

1
1 o
o
<f> this b e c o m e s

1965

L E T T E R TO THE EDITOR

73

Again since for the original Fibonacci sequence <f> is differentlydefined (in t e r m s of your f's) we get D = -1 in this cas% on m y definition but this is not disconcerting,, To m y mind the original Fibonacci
sequence is sufficiently e x t r a o r d i n a r y (on c o m p a r i s o n with other such
sequences) that it d e s e r v e s a D with a sign different from that of the
o t h e r s . Inspection of the D's as p r e s e n t e d in your p a p e r leads m e to
the following c o n j e c t u r e s (I am inclined to think that (a) i t i s not difficult
to prove them, (b) it h a s been done so before - thus I have not taken
the t r o u b l e ) .
Let p r i m e n u m b e r s of the form
We have then
(1)

The set of feasible

lOn 1 be r e p r e s e n t e d by g-, .

D's is m a d e up of - 1 , 5, all g ^ ' s , all p r o d -

u c t s of g i J s which we denote by

(i.e.

= g.g.. . . g j j

all

n u m b e r s of the form 5gk> and a l l n u m b e r s of the form 5Q . In o t h e r


w o r d s a n e c e s s a r y condition for an i n t e g e r to be a D i s that it belongs

to the s&TJ^TTT,
(2)
D's.

g k , Q m , 5g k , 5 Q m } .

Each n u m b e r in the above set m a y indeed be found in the l i s t of


In other w o r d s , the above is a l s o a sufficient condition.

(3)
T h e n u m b e r of s e q u e n c e s a s s o c i a t e d w i t h a g i v e n v a l u e of D i s
s i m p l y r e l a t e d to its f a c t o r i z a t i o n p r o p e r t i e s . I r e s e r v e a final f o r m u l a t i o n of m y c o n j e c t u r e o n t h i s p a r t u n t i l I h a v e s e e n m o r e " e x p e r i m e n t a l m a t e r i a l , " i . e . , a t a b l e of D ' s ( w i t h a s s o c i a t e d s e q u e n c e s )
b e t w e e n 1000 a n d 2 0 0 0 . It i s a l r e a d y o b v i o u s t h a t f o r - 1 a n d 5 w e g e t
t h e s e l f - d u a l s e q u e n c e s a n d f o r e a c h g k a n d 5 g k w e h a v e o n e p a i r of
a
dual s e q u e n c e s . As for a Q
it i s o b v i o u s t h a t if i t e q u a l s g (a > 1)
m
Ac
w e h a v e a g&a i n o n e a s s o c i a t e d /p a i r , b u t f o r t h e c a s e Q m =toig a g& j. . . . a kP
t h e n u m b e r of a s s o c i a t e d p a i r s i s a f u n c t i o n of t h e d e g r e e of " c o m p o s i t e n e s s " and this should be looked into a little m o r e c a r e f u l l y by
m e a n s of a n e x t e n d e d T a b l e . F i n a l l y , t h e n u m b e r of p a i r s of F i b o n a c c i
s e q u e n c e s a s s o c i a t e d w i t h 5Q
i s i d e n t i c a l w i t h t h e n u m b e r of p a i r s
associated with Q ,
m
If y o u a r e a w a r e of l i t e r a t u r e r e l a t i n g t o t h e s e c o n j e c t u r e s , k i n d l y
l e t m e k n o w . A l s o if y o u h a v e a n e x t e n d e d t a b l e of t h e D ' s I s h o u l d
a p p r e c i a t e a copy*
I h o p e s o m e of m y r e m a r k s m a y h a v e b e e n of u s e f o r o r d e r i n g
and c l a s s i f i c a t i o n p u r p o s e s .

xxxxxxxxxxxxxxx

ELEMENTARY PROBLEMS AND SOLUTIONS


Edited by A.P. HILLMAN
University of Santa Clara, Santa Clara, California

Send all communications regarding Elementary Problems and


Solutions to Professor A. P. Hillman, Mathematics Department, University of Santa Clara,

Santa Clara,

California.

Any problem be-

lieved to be new in the area of recurrent sequences and any new approaches to existing problems will be welcomed.

The proposer should

submit each problem with solution in legible form, preferably typed in


double spacing with name and address of the proposer as a heading.
Solutions to problems listed below should be submitted on separate signed sheets within two months of publication.
B-58

Proposed by Sidney Kravitz, Dover, New Jersey

Show that no Fibonacci number other than 1, 2, or 3 is equal to


^ Lucas number.
B-59

Proposed by Brother V. Alfred, St. Mary's College,

California

Show that the volume of a truncated right circular cone of slant


height
&

with

, and F . , the diameters of the bases is


n-1
n+1
^

B-60

n-1>/24

Proposed by Verner E. Hoggatt, Jr., San Jose State College, San Jose,
2

Show that

L0 L LO - 5F 0 , , = 1, where F
and
2n 2n+2
2n+l
n
n-th Fibonacci number and Lucas number,respectively.
B-61

L are the
n

Proposed by J. A. H. Hunter, Toronto, Ontario

Define a sequence

U,, U?, . . .

by U, = 3 and

U = U , + n 2 + n + 1 for
n
n-1
Prove that
B-62

California

n >1

U = 0 (mod n) if ng 0 (mod 3).

Proposed by Brother U. Alfred, St. Mary's College,

California

Prove that a Fibonacci number with odd subscript cannot be represented as the sum of squares of two Fibonacci numbers in more than
one way.
74

1965

E L E M E N T A R Y P R O B L E M S AND SOLUTIONS

B - 6 3 An old problem whose source is unknown, suggested

75

by Sidney Kravitz, Dover, New Jersey

In A ABC let s i d e s AB and AC be equal. Let t h e r e be


point D on side AB such that AD = CD = BC. Show that
2cos 3-A = A B / B C = (1 +

N/5)/2

.,

the golden m e a n .
SOLUTIONS
A BOUND ON BOUNDED FIBONACCI NUMBERS
B-44

Proposed by Douglas Lind, Falls Church,

Virginia

P r o v e that for e v e r y positive i n t e g e r k t h e r e a r e no m o r e than


k
k+1
n F i b o n a c c i n u m b e r s between n and n
Solution by the proposer.

A s s u m e the m a x i m u m ,
(1)
x
'
Now

nk < F

F ,9, . . . , F , < nk+1


r+2
r+n

xl,

r+1

n-1
**

r+n-1
r+j

**
=

P.- 2
r+2

r+n+l

But by (1),
n-1
2**

, + F r+2
,7 >n-nk
r+j

and hence
r+n+1

thus proving the p r o p o s i t i o n .

k+1

F.

76

ELEMENTARY PROBLEMS AND SOLUTIONS

February

ANOTHER SUM
B-45

Proposed by Charles R. Wall, Texas Christian University,

Ft. Worth, Texas

Let H

be the n - t h teg e n e r a l i z e d Fibonacci n u m b e r , i. e. , let


n
H,1 and H~2 be a r b i t r a r y7 and H n+2
IO = H ., + H
n+1
n for n > 0. Show
+ Hn = Hn+4
that nHj + ( n - l ) H 2 + (n--2)H 3 +
J.-(n+2)H0-H1 ,
' 2 1
Solution by David Zeitlin,

Minneapolis,

Minnesota.

In B-20 (see Fibonacci Q u a r t e r l y ,

2(1964) p. 77), it was shown

that
n
1

= H

n+2 '

'

j=l
In B-40 (see Fibonacci Q u a r t e r l y , 2(1964), p . 155), Wall p r o p o s e d that

. . . + H, + H ,
JH. = (n + l)Hn+2_ - H n+4
1
2

Thus, the d e s i r e d sum


n

[(n+1) - j] H.. =

(n+1) 2 H. - 2

j=l

j=l

JH.

j=l

[(n+l)Hn+2 - (n+l)H2]- [(n+l)Hn+2-Hn+4+H1+H2]


=

H n + 4 - (n+2)H 2 - Hj

Also solved by Douglas Lind, Kenneth E. Newcomer, Farid K. Shuayto, Sheryl B.


Howard L. Walton, Charles Zeigenfus, and the proposer.

A CONTINUANT
B-46

DETERMINANT

Proposed by C.A. Church, Jr., Duke University,

Durham, North Carolina

Evaluate the n - t h o r d e r d e t e r m i n a n t

a+b

ab

a+b

ab

a+b

ab

= 1 0

a+b

Tadlock,

1965

ELEMENTARY PROBLEMS AND SOLUTIONS

Solution by F.D. Parker, SUNY, Buffalo,

11

N.Y.

We denote the value of the d e t e r m i n a n t of o r d e r n by D(n), and


2
2
notice that D(l) = a + b and D(2) = a + ab + b . Expanding D(n) by
the f i r s t row, we see that
D(n) = (a + b) D ( n - l ) - ab D(n-2)

This is a homogeneous linear second o r d e r difference equation;


if a T* b, the solution which fits the initial conditions is
D(n) = ( a n + 1 - b n + 1 ) / ( a - b )

..

If a = b, the solution which fits the initial conditions is D(n) = (1 + n) a .


Also solved by Joel L. Brenner, Douglas Lind, C.B.A. Peck, David Zeitlin,

and the proposer.

Lind, Peck, and Zeitlin pointed out that B-46 is a s p e c i a l c a s e of B - 1 3 .


P e c k a l s o n o t e d that B-46 is an e x a m p l e of a c l a s s of continuants m e n tioned by J. J. S y l v e s t e r in the P h i l o s o p h i c a l Magazine,
(1853)446-457.

S e r i e s 4, 5

(See T. Muir, H i s t o r y of the T h e o r y of D e t e r m i n a n t s

(Dover) Vol. I, p . 418,)

B r e n n e r noted that B-46 and s i m i l a r p r o b -

l e m s occur as Nos. 217, 225, 234, e t c . in F a d d e e v and Sominski,


P r o b l e m s in Higher Algebra,

a t r a n s l a t i o n of which will soon be pub-

lished by W. H. F r e e m a n .
CONSECUTIVE COMPOSITE FIBONACCI NUMBERS
B-47

Proposed by Barry Litvack,

University of Michigan, Ann Arbor, Michigan

P r o v e that for e v e r y positive i n t e g e r

k there are

consecutive

F i b o n a c c i n u m b e r s each of which is c o m p o s i t e .
Solution by Sidney Kravitz, Dover, New Jersey

Let F

for

be the n - t h F i b o n a c c i n u m b e r . We note that F > 1


n
n
n > 2, that F . divides F . and that j is a d i v i s o r of (k+2)! + j

for

3 i j 1 k+2.
F

Thus the k consecutive F i b o n a c c i n u m b e r s

(k+2)!+3'

a r e divisible by F~, F,9

(k+2)I + 4 ' " "


. , , F,

(k+2)I + k + 2

respectively.

Also solved by R.W. Castown, Douglas Lind, F.D. Parker, and the proposer.

78

ELEMENTARY PROBLEMS AND SOLUTIONS

February

A BINOMIAL EXPANSION
B-48

Proposed by H.H. Ferns, University of Victoria, Victoria, British Columbia, Canada

Prove that
r " 1

r
(-2 F

2
,

(-Z)\)Fk=

where

if

is an even positive integer


/ 2
/

v 2 F -2(5)
F ,? = F

in which the

F,

if

r is an odd positive integer,

. + F (F ] = F~ = 1) and find the corresponding sum


are replaced by the Lucas numbers

Solution by D.G. Mead, University of Santa Clara, Santa Clara,

Let S be the given sum.


Fn

L, .

California

By the Binet formula,

= (a11 - b n )/(a-b)

where a = (1 + /5)/2 and b = 1 - a.

Then a - b = y/fT = 1 - 2a = 2b - l f

and

S + (-2) r F r

(-2)kFk

k=0
r

= 2 0 C(-2a>k " <- 2b ) k ]


=

(1 - 2a) r - (1 - 2b) r

_ ( / 5 ) r [1 - ( - l ) r ]

The desired conclusion follows immediately.


Similarly one sees from
sum for the Lucas num bers is
-2+2

L for
r

r odd

Also solved by the proposer.

L =a
r

-2 - 2 L

+b

that the corresponding

+ 2( \/ 5)T for

even and

1 965

ELEMENTARY PROBLEMS AND SOLUTIONS

79

AN ALPHAMETIC
B-49

Proposed by Anton Glaser, Pennsylvania

State University,

Abington,

Pennsylvania

Let <fi r e p r e s e n t the l e t t e r "oh".


Given that T, W, 0 , L, V, P , and TW< a r e

TW0

F i b o n a c c i n u m b e r s , solve the c r y p t a r i t h m

THE

IS

in the base 14, introducing the digits

0NLY

/3, Y , and 5 in base 14 for 10, 11,

EVEN

12, and 1 3 in base 10.


Solution by Charles Ziegenfus,

PRIME
Madison College, Harrisonburg,

Virginia

With a little calculation one o b s e r v e s that the Fibonacci n u m b e r


c o r r e s p o n d i n g to TW< is 2584.
L = 3 or 5, and V = 3 or 5

Thus,

Next we note that 8 + E + (2 or 3) = 1R,

so that E = 4 + R or E = 3 + R0

Tabulating t h e s e r e s u l t s :

Further, 8 + S + E + Y + N = k E
in base 14.

T = , W = 2, ' <f> = 8, P = 1,

or S + Y + N = k 0 - 8 = 6, 16, or 26

T h e r e a r e no p o s s i b l e choices for

S + Y + N = 6 or 26.
( 4, 7, 9 ) ; {4, 6, a }

Thus

S, Y, N

S, Y, N such that

can be chosen from (0, 9 , $ } ;

Tabulating this with the p r e v i o u s r e s u l t we ob-

tain:
R

S, Y, N

4, 7, 9
or
0, 9, /3

4, 6, a

0, 9, 0

0, 9, 0

4, 6, a

Further,
T + W+0+I + S + T+H + E + 0 + N + L + Y + E+V + E + N
- P - R - I - M - E

is a multiple of 8 .

We r e d u c e the above to 6 + (2 E - R) + H + N - M = S k,
On substituting the p o s s i b l e values for
duce this p r o b l e m to the following c a s e s :

R and E we f u r t h e r r e -

80

ELEMENTARY PROBLEMS AND SOLUTIONS

a.

R = 6 and E = a,

7 + N + H - M = 8' k.

b.

R = 7 and E = 0 ,

8 + N + H - M = S"k.

c.

R = 4 and E = 7,

3 + N + H - M = 8' k.

d.

R = 7 and E = a,
R = 9 and E = y9

6 + N + H - M .= S ' k .
8 + H + N - M = 8'k0

February

e.
F r o m the p r e v i o u s table we o b s e r v e that t h e r e a r e exactly t h r e e choices
for

N.

Using t h e s e in the above c a s e s r e d u c e s the p r o b l e m to an

equation involving only H and M and only t h r e e choices for t h e s e .


Thus we obtain two distinct solutions (actually four since S and Y
can be i n t e r c h a n g e d ) .
0

S or Y

S or Y

S or Y

<P
<t> S

or Y

Also solved by the proposer and partially solved by J.A.H.

Hunter.

AND A N O T H E R SUM
B-50

Proposed by Douglas Lind, Falls Church,

Virginia

P r o v e that
n

T2F2 - ( n )F. 1 = F 2 ,

j=0
Solution by David 7.eitlin, Minneapolis,

Minnesota.

Since
n

F7 = F F , . ,
j
n n+r

j=0
X

(s n ) F . = F 9 = F L = F (F ,, + F . ),
y
j
Zn
n n
n n+1
n-1

j=o
the d e s i r e d sum is
2 F F ,, - F F ,, - F F " = F (F ,. - F . ) = F 2
n n+1
n n+1
n n-1
n n+1
n-1'
n
Also solved by H.H. Ferns, Farid K.Shuayto, Sheryl B. Tadlock,

XXXXXXXXXXXXXXX

and the proposer.

S-ar putea să vă placă și